SlideShare a Scribd company logo
1 of 174
2-1 
Operations Research 
Books: 
Operations Research : An introduction 
by Hamdy A Taha Ninth ed Pearson
2-2 
Today, optimization methods are used 
everywhere in business, industry, 
government, engineering, and computer 
science. 
You will find optimization taught in various 
departments of engineering, computer 
science, business, mathematics, and 
economics;
The desire for optimality (perfection) is inherent 
for humans 
Optimization techniques used today have trace their 
origins during World War II to improve the 
effectiveness of the war efforts 
For example: 
What is the optimum allocation of gasoline supplies 
among competing campaigns? What is the best search 
and bombing pattern for anti-submarine patrols? How 
to manage in the best way the limited food. 
2-3
2-4 
Here are a few more examples of optimization 
problems: 
·How should the transistors and other devices be laid 
out in a new computer chip so that the layout takes up 
the least area? 
·What is the smallest number of warehouses, and 
where should they be located so that the maximum 
travel time from any retail sales outlet to the closest 
warehouse is less than 6 hours? 
·How should telephone calls be routed between two 
cities to permit the maximum number of simultaneous 
calls?
2-5
2-6 
A schematic view of 
modeling/optimization process 
Real-world 
problem 
Mathematical 
model 
Solution to 
model 
Solution to 
real-world 
problem 
assumptions, 
abstraction, data 
simplifications 
optimization 
algorithm 
interpretation 
makes sense? change the 
model, assumptions?
2-7 
Types of 
Optimization Models 
Stochastic 
(probabilistic 
information on data) 
Deterministic 
(data are certain) 
Discrete, Integer 
(S = Zn) 
Continuous 
(S = Rn) 
Linear 
(f and g are linear) 
Nonlinear 
(f and g are nonlinear)
2-8 
What is Optimization process? 
 Optimization is an iterative process by which a desired solution 
(max/min) of the problem can be found while satisfying all its 
constraint or bounded conditions. 
 Optimization problem could be linear or non-linear. 
 The search procedure is termed as 
algorithm.
2-9 
Mathematical models in 
Optimization 
 The general form of an optimization model: 
min or max f(x1,…,xn) (objective function) 
subject to gi(x1,…,xn) ≥ 0 (functional constraints) 
x1,…,xn Î S (set 
constraints) 
 x1,…,xn are called decision variables 
 In words, the goal is to find 
 --- x1,…,xn that satisfy the constraints; 
--- achieve min (max) objective function value.
2-10 
Optimization Problems 
Page 10 
Unconstrained optimization 
min{f(x) : x Î Rn} 
Constrained optimization 
min{f(x) : ci(x) ≤ 0, i Î I, cj(x) = 0, j Î E} 
Quadratic programming 
Tx ≤ bi, i Î I, ai 
min{1/2xTQx + cTx : ai 
Tx = bj, j Î E} 
Zero-One programming 
min{cTx : Ax = b, x Î {0,1}n, c Î Rn, b Î Rm} 
Mixed Integer Programming 
min{cTx : Ax ≤ b, x ≥ 0, xi Î Zn, i Î I, xr Î Rn, r Î R}
2-11 
Linear programming is the most widely 
used of the major techniques for 
constrained optimization. 
“Programming” here does not imply 
computer programming –it is an old word 
for “planning”. 
In linear programming, all of the 
underlying models of the real-world 
processes are linear, hence you can think of 
“linear programming” as “planning using 
linear models”
2-12 
= £ å= 
a x b i m 
( 1, 2, ..., ) 
1 
x j n 
12 
Introduction 
 Problems of this kind are called “linear programming problems” or “LP 
problems” for short; linear programming is the branch of applied 
mathematics concerned with these problems. 
 A linear programming problem is the problem of maximizing (or minimizing) a 
linear function subject to a finite number of linear constraints. 
 Standard form: 
å= n 
j 
j j c x 
1 
0 ( 1, 2, ..., ) 
j 
i 
n 
j 
ij j 
³ = 
maximize 
subject to
2-13 
History of Linear Programming 
13 
 It started in 1947 when G.B.Dantzig design 
the “simplex method” for solving linear 
programming formulations of U.S. Air 
Force planning problems. 
 It soon became clear that a surprisingly 
wide range of apparently unrelated 
problems in production management could 
be stated in linear programming terms and 
solved by the simplex method. 
 Later, it was used to solve problems of 
management. It’s algorithm can also used to 
network flow problems.
 On Oct.14th,1975, the Royal Sweden Academy of Science awarded the 
Nobel Prize in economic science to L.V.Kantorovich and 
T.C.Koopmans ”for their contributions to the theory of optimum 
allocation of resources” 
2-14 
History of Linear Programming 
 The breakthrough in looking for a theoretically satisfactory algorithm 
to solve LP problems came in 1979 when L.G.Khachian published a 
description of such an algorithm. 
14
2-15 
15 
Applications 
 Efficient allocation of scarce resources 
 diet problem 
 Scheduling production and inventory 
 multistage scheduling problems 
 The cutting-stock problem 
 find a way to cut paper or textiles rolls by 
complicated summary of orders 
 Approximating data by linear functions 
 find approximate solutions to possibly 
unsolvable systems of linear equations
2-16 
 Model Formulation 
 A Maximization Model Example 
 Graphical Solutions of Linear Programming Models 
 A Minimization Model Example 
 Irregular Types of Linear Programming Models 
 Characteristics of Linear Programming Problems
Linear Programming: An Overview 
 Objectives of business decisions frequently involve 
maximizing profit or minimizing costs. 
 Linear programming uses linear algebraic relationships 
to represent a firm’s decisions, given a business objective, 
and resource constraints. 
 Steps in application: 
1. Identify problem as solvable by linear programming. 
2. Formulate a mathematical model of the unstructured 
2-17 
problem. 
3. Solve the model. 
4. Implementation
2-18 
Properties of Linear Programming Models 
• Proportionality - The rate of change (slope) of the objective 
function and constraint equations is constant. 
• Additivity - Terms in the objective function and constraint 
equations must be additive. 
• Divisibility -Decision variables can take on any fractional value 
and are therefore continuous as opposed to integer in nature. 
• Certainty - Values of all the model parameters are assumed to 
be known with certainty (non-probabilistic).
2-19 
Model Components 
 Decision variables - mathematical symbols representing levels 
of activity of a firm. 
 Objective function - a linear mathematical relationship 
describing an objective of the firm, in terms of decision 
variables - this function is to be maximized or minimized. 
 Constraints – requirements or restrictions placed on the firm by 
the operating environment, stated in linear relationships of the 
decision variables. 
 Parameters - numerical coefficients and constants used in the 
objective function and constraints.
2-20 
Summary of Model Formulation Steps 
Step 1 : Clearly define the decision variables 
Step 2 : Construct the objective function 
Step 3 : Formulate the constraints
2-21 
21 
Introduction 
 A Diet Problem 
eg: Polly wonders how much money she must spend on food in order to get 
all the energy (1,000 kcal ), protein (55 g), and calcium (800 mg) that she 
needs every day. She choose six foods that seem to be cheap sources of 
the nutrients: 
Food Serving 
size 
Energy 
(kcal) 
Protein (g) Calcium 
(mg) 
Price per 
serving (c) 
Oatmeal 28 g 110 4 2 3 
Chicken 100 g 205 32 12 24 
Eggs 2 large 160 13 54 13 
Whole Milk 237 cc 160 8 285 9 
Cherry pie 170 g 420 4 22 20 
Pork with 
beans 
260 g 260 14 80 19
2-22 
22 
Introduction 
 Servings-per-day limits on all six foods: 
Oatmeal at most 4 servings per day 
Chicken at most 3 servings per day 
Eggs at most 2 servings per day 
Milk at most 8 servings per day 
Cherry pie at most 2 servings per day 
Pork with beans at most 2 servings per day 
 Now there are so many combinations seem promising that one 
could go on and on, looking for the best one. Trial and error is 
not particularly helpful here.
2-23 
1 2 3 4 5 6 minimize 3x + 24x + 13x + 9x + 20x + 19x 
subject to 
x x x x x x 
+ + + + + ³ 
110 205 160 160 420 260 1,000 
1 2 3 4 5 6 
x x x x x x 
+ + + + + ³ 
4 32 13 8 4 14 55 
1 2 3 4 5 6 
x x x x x x 
+ + + + + ³ 
23 
Introduction 
 A new way to express this—using inequalities: 
x 
£ £ 
0 4 
1 
x 
£ £ 
0 3 
2 
x 
£ £ 
0 2 
3 
£ £ 
0 8 
4 
x 
£ £ 
0 2 
5 
x 
£ £ 
0 2 
6 
x 
2 12 54 285 22 80 800 
1 2 3 4 5 6
2-24 
= £ å= 
a x b i m 
( 1, 2, ..., ) 
1 
x j n 
24 
Introduction 
 Problems of this kind are called “linear programming problems” or “LP 
problems” for short; linear programming is the branch of applied 
mathematics concerned with these problems. 
 A linear programming problem is the problem of maximizing (or minimizing) a 
linear function subject to a finite number of linear constraints. 
 Standard form: 
å= n 
j 
j j c x 
1 
0 ( 1, 2, ..., ) 
j 
i 
n 
j 
ij j 
³ = 
maximize 
subject to
2-25 
LP Model Formulation 
A Maximization Example (1 of 4) 
 Product mix problem - Beaver Creek Pottery Company 
 How many bowls and mugs should be produced to maximize 
profits given labor and materials constraints? 
 Product resource requirements and unit profit: 
Resource Requirements 
Product 
Labor 
(Hr./Unit) 
Clay 
(Lb./Unit) 
Profit 
($/Unit) 
Bowl 1 4 40 
Mug 2 3 50
2-26 
LP Model Formulation 
A Maximization Example (2 of 4)
2-27 
LP Model Formulation 
A Maximization Example (3 of 4) 
Resource 40 hrs of labor per day 
Availability: 120 lbs of clay 
Decision x1 = number of bowls to produce per day 
Variables: x2 = number of mugs to produce per day 
Objective Maximize Z = $40x1 + $50x2 
Function: Where Z = profit per day 
Resource 1x1 + 2x2 £ 40 hours of labor 
Constraints: 4x1 + 3x2 £ 120 pounds of clay 
Non-Negativity x1 ³ 0; x2 ³ 0 
Constraints:
2-28 
LP Model Formulation 
A Maximization Example (4 of 4) 
Complete Linear Programming Model: 
Maximize Z = $40x1 + $50x2 
subject to: 1x1 + 2x2 £ 40 
4x1 + 3x2 £ 120 
x1, x2 ³ 0
Feasible Solutions 
A feasible solution does not violate any of the constraints: 
Example: x1 = 5 bowls 
2-29 
x2 = 10 mugs 
Z = $40x1 + $50x2 = $700 
Labor constraint check: 1(5) + 2(10) = 25 < 40 hours 
Clay constraint check: 4(5) + 3(10) = 50 < 120 pounds
2-30 
Infeasible Solutions 
An infeasible solution violates at least one of the 
constraints: 
Example: x1 = 10 bowls 
x2 = 20 mugs 
Z = $40x1 + $50x2 = $1400 
Labor constraint check: 1(10) + 2(20) = 50 > 40 hours
2-31 
Graphical Solution of LP Models 
 Graphical solution is limited to linear programming models 
containing only two decision variables (can be used with three 
variables but only with great difficulty). 
 Graphical methods provide visualization of how a solution for 
a linear programming problem is obtained.
Coordinate Axes 
Graphical Solution of Maximization Model (1 of 12) 
2-32 
Maximize Z = $40x1 + $50x2 
subject to: 1x1 + 2x2 £ 40 
4x1 + 3x2 £ 120 
x1, x2 ³ 0 
Figure Coordinates for Graphical Analysis 
X1 is bowls 
X2 is mugs
Labor Constraint 
Graphical Solution of Maximization Model (2 of 12) 
2-33 
Figure Graph of Labor Constraint 
Maximize Z = $40x1 + $50x2 
subject to: 1x1 + 2x2 £ 40 
4x1 + 3x2 £ 120 
x1, x2 ³ 0
Labor Constraint Area 
Graphical Solution of Maximization Model (3 of 12) 
2-34 
Figure Labor Constraint Area 
Maximize Z = $40x1 + $50x2 
subject to: 1x1 + 2x2 £ 40 
4x1 + 3x2 £ 120 
x1, x2 ³ 0
Clay Constraint Area 
Graphical Solution of Maximization Model (4 of 12) 
2-35 
Figure Clay Constraint Area 
Maximize Z = $40x1 + $50x2 
subject to: 1x1 + 2x2 £ 40 
4x1 + 3x2 £ 120 
x1, x2 ³ 0
Both Constraints 
Graphical Solution of Maximization Model (5 of 12) 
2-36 
Maximize Z = $40x1 + $50x2 
subject to: 1x1 + 2x2 £ 40 
4x1 + 3x2 £ 120 
x1, x2 ³ 0 
Figure Graph of Both Model Constraints
Feasible Solution Area 
Graphical Solution of Maximization Model (6 of 12) 
2-37 
Figure Feasible Solution Area 
Maximize Z = $40x1 + $50x2 
subject to: 1x1 + 2x2 £ 40 
4x1 + 3x2 £ 120 
x1, x2 ³ 0
Objective Function Solution = $800 
Graphical Solution of Maximization Model (7 of 12) 
2-38 
Maximize Z = $40x1 + $50x2 
subject to: 1x1 + 2x2 £ 40 
4x1 + 3x2 £ 120 
x1, x2 ³ 0 
Figure Objection Function Line for Z = $800
2-39 
Alternative Objective Function Solution Lines 
Graphical Solution of Maximization Model (8 of 12) 
Maximize Z = $40x1 + $50x2 
subject to: 1x1 + 2x2 £ 40 
4x1 + 3x2 £ 120 
x1, x2 ³ 0 
Figure Alternative Objective Function Lines
Optimal Solution 
Graphical Solution of Maximization Model (9 of 12) 
2-40 
Maximize Z = $40x1 + $50x2 
subject to: 1x1 + 2x2 £ 40 
4x1 + 3x2 £ 120 
x1, x2 ³ 0 
Figure Identification of Optimal Solution Point
2-41 
Optimal Solution Coordinates 
Graphical Solution of Maximization Model (10 of 12) 
Figure Optimal Solution Coordinates 
Maximize Z = $40x1 + $50x2 
subject to: 1x1 + 2x2 £ 40 
4x1 + 3x2 £ 120 
x1, x2 ³ 0
2-42 
Extreme (Corner) Point Solutions 
Graphical Solution of Maximization Model (11 of 12) 
Figure Solutions at All Corner Points 
Maximize Z = $40x1 + $50x2 
subject to: 1x1 + 2x2 £ 40 
4x1 + 3x2 £ 120 
x1, x2 ³ 0
2-43 
Optimal Solution for New Objective Function 
Graphical Solution of Maximization Model (12 of 12) 
Maximize Z = $70x1 + $20x2 
subject to: 1x1 + 2x2 £ 40 
4x1 + 3x2 £ 120 
x1, x2 ³ 0 
Figure Optimal Solution with Z = 70x1 + 20x2
 Standard form requires that all constraints be in the form 
of equations (equalities). 
 A slack variable is added to a £ constraint (weak 
inequality) to convert it to an equation (=). 
 A slack variable typically represents an unused resource. 
 A slack variable contributes nothing to the objective 
function value. 
2-44 
Slack Variables
Linear Programming Model: Standard Form 
2-45 
Max Z = 40x1 + 50x2 + 0s1 + 0s2 
subject to:1x1 + 2x2 + s1 = 40 
4x1 + 3x2 + s2 = 120 
x1, x2, s1, s2 ³ 0 
Where: 
x1 = number of bowls 
x2 = number of mugs 
s1, s2 are slack variables 
Figure Solution Points A, B, and C with Slack
2-46 
LP Model Formulation – Minimization (1 of 8) 
 Two brands of fertilizer available - Super-gro, Crop-quick. 
 Field requires at least 16 pounds of nitrogen and 24 pounds of 
phosphate. 
 Super-gro costs $6 per bag, Crop-quick $3 per bag. 
 Problem: How much of each brand to purchase to minimize total 
cost of fertilizer given following data ? 
Chemical Contribution 
Brand Nitrogen 
(lb/bag) 
Phosphate 
(lb/bag) 
Super-gro 2 4 
Crop-quick 4 3
2-47 
LP Model Formulation – Minimization (2 of 8) 
Figure Fertilizing 
farmer’s field
2-48 
LP Model Formulation – Minimization (3 of 8) 
Decision Variables: 
x1 = bags of Super-gro 
x2 = bags of Crop-quick 
The Objective Function: 
Minimize Z = $6x1 + 3x2 
Where: $6x1 = cost of bags of Super-Gro 
$3x2 = cost of bags of Crop-Quick 
Model Constraints: 
2x1 + 4x2 ³ 16 lb (nitrogen constraint) 
4x1 + 3x2 ³ 24 lb (phosphate constraint) 
x1, x2 ³ 0 (non-negativity constraint)
2-49 
Constraint Graph – Minimization (4 of 8) 
Minimize Z = $6x1 + $3x2 
subject to: 2x1 + 4x2 ³ 16 
4x1 + 3x2 ³ 24 
x1, x2 ³ 0 
Figure Graph of Both Model Constraints
Feasible Region– Minimization (5 of 8) 
Figure Feasible Solution Area 
Minimize Z = $6x1 + $3x2 
subject to: 2x1 + 4x2 ³ 16 
4x1 + 3x2 ³ 24 
x1, x2 ³ 0 
Copyright © 2010 Pearson Education, Inc. Publishing as Prentice Hall 2-50
2-51 
Optimal Solution Point – Minimization (6 of 8) 
Figure Optimum Solution Point 
Minimize Z = $6x1 + $3x2 
subject to: 2x1 + 4x2 ³ 16 
4x1 + 3x2 ³ 24 
x1, x2 ³ 0
Surplus Variables – Minimization (7 of 8) 
 A surplus variable is subtracted from a ³ constraint to 
convert it to an equation (=). 
 A surplus variable represents an excess above a 
constraint requirement level. 
 A surplus variable contributes nothing to the calculated 
value of the objective function. 
 Subtracting surplus variables in the farmer problem 
constraints: 
2-52 
2x1 + 4x2 - s1 = 16 (nitrogen) 
4x1 + 3x2 - s2 = 24 (phosphate)
2-53 
Graphical Solutions – Minimization (8 of 8) 
Minimize Z = $6x1 + $3x2 + 0s1 + 0s2 
subject to: 2x1 + 4x2 – s1 = 16 
4x1 + 3x2 – s2 = 24 
x1, x2, s1, s2 ³ 0 
Figure Graph of Fertilizer Example
Irregular Types of Linear Programming Problems 
2-54 
For some linear programming models, the general rules 
do not apply. 
 Special types of problems include those with: 
 Multiple optimal solutions 
 Infeasible solutions 
 Unbounded solutions
2-55 
Multiple Optimal Solutions Beaver Creek Pottery 
The objective function is 
parallel to a constraint line. 
Maximize Z=$40x1 + 30x2 
subject to: 1x1 + 2x2 £ 40 
4x1 + 3x2 £ 120 
x1, x2 ³ 0 
Where: 
x1 = number of bowls 
x2 = number of mugs 
Figure Example with Multiple Optimal Solutions
2-56 
An Infeasible Problem 
Every possible solution 
violates at least one constraint: 
Maximize Z = 5x1 + 3x2 
subject to: 4x1 + 2x2 £ 8 
Figure Graph of an Infeasible Problem 
x1 ³ 4 
x2 ³ 6 
x1, x2 ³ 0
2-57 
An Unbounded Problem 
Value of the objective 
function increases indefinitely: 
Maximize Z = 4x1 + 2x2 
subject to: x1 ³ 4 
Figure Graph of an Unbounded Problem 
x2 £ 2 
x1, x2 ³ 0
2-58 
Characteristics of Linear Programming Problems 
 A decision amongst alternative courses of action is required. 
 The decision is represented in the model by decision variables. 
 The problem encompasses a goal, expressed as an objective 
function, that the decision maker wants to achieve. 
 Restrictions (represented by constraints) exist that limit the 
extent of achievement of the objective. 
 The objective and constraints must be definable by linear 
mathematical functional relationships.
2-59 
Properties of Linear Programming Models 
 Proportionality - The rate of change (slope) of the objective 
function and constraint equations is constant. 
 Additivity - Terms in the objective function and constraint 
equations must be additive. 
 Divisibility -Decision variables can take on any fractional value 
and are therefore continuous as opposed to integer in nature. 
 Certainty - Values of all the model parameters are assumed to 
be known with certainty (non-probabilistic).
2-60 
Problem Statement 
Example Problem No. 1 (1 of 3) 
■ Hot dog mixture in 1000-pound batches. 
■ Two ingredients, chicken ($3/lb) and beef ($5/lb). 
■ Recipe requirements: 
at least 500 pounds of “chicken” 
at least 200 pounds of “beef” 
■ Ratio of chicken to beef must be at least 2 to 1. 
■ Determine optimal mixture of ingredients that will 
minimize costs.
2-61 
Solution 
Example Problem No. 1 (2 of 3) 
Step 1: 
Identify decision variables. 
x1 = lb of chicken in mixture 
x2 = lb of beef in mixture 
Step 2: 
Formulate the objective function. 
Minimize Z = $3x1 + $5x2 
where Z = cost per 1,000-lb batch 
$3x1 = cost of chicken 
$5x2 = cost of beef
2-62 
Solution 
Example Problem No. 1 (3 of 3) 
Step 3: 
Establish Model Constraints 
x1 + x2 = 1,000 lb 
x1 ³ 500 lb of chicken 
x2 ³ 200 lb of beef 
x1/x2 ³ 2/1 or x1 - 2x2 ³ 0 
x1, x2 ³ 0 
The Model: Minimize Z = $3x1 + 5x2 
subject to: x1 + x2 = 1,000 lb 
x1 ³ 500 
x2 ³ 200 
x1 - 2x2 ³ 0 
x,x³ 0
2-63 
Solve the following model 
graphically: 
Maximize Z = 4x1 + 5x2 
subject to: x1 + 2x2 £ 10 
6x1 + 6x2 £ 36 
x1 £ 4 
x1, x2 ³ 0 
Step 1: Plot the constraints 
as equations 
Figure Constraint Equations
2-64 
Maximize Z = 4x1 + 5x2 
subject to: x1 + 2x2 £ 10 
6x1 + 6x2 £ 36 
x1 £ 4 
x1, x2 ³ 0 
Step 2: Determine the feasible 
solution space 
Figure Feasible Solution Space and Extreme Points
2-65 
Maximize Z = 4x1 + 5x2 
subject to: x1 + 2x2 £ 10 
6x1 + 6x2 £ 36 
x1 £ 4 
x1, x2 ³ 0 
Step 3 and 4: Determine the 
solution points and optimal 
solution 
Figure Optimal Solution Point
2-66 
Simplex Method 
 SSiimmpplleexx MMeetthhoodd 
 CChhaarraacctteerriissttiiccss ooff SSiimmpplleexx MMeetthhoodd 
 WWhhyy wwee sshhoouulldd ssttuuddyy tthhee SSiimmpplleexx MMeetthhoodd?? 
 SSuummmmaarryy ooff tthhee SSiimmpplleexx MMeetthhoodd 
 EExxaammpplleess ssoollvveedd bbyy ccoonndduuccttiinngg ttaabbuullaarr mmeetthhoodd 
66
SIMPLEX METHOD 
 Linear programming mmooddeellss ccoouulldd bbee ssoollvveedd 
aallggeebbrraaiiccaallllyy.. 
 TThhee mmoosstt wwiiddeellyy uusseedd aallggeebbrraaiicc pprroocceedduurree ffoorr ssoollvviinngg 
lliinneeaarr pprrooggrraammmmiinngg pprroobblleemm iiss ccaalllleedd tthhee SSiimmpplleexx 
MMeetthhoodd.. 
 TThhee ssiimmpplleexx mmeetthhoodd iiss aa ggeenneerraall--ppuurrppoossee lliinneeaarr-- 
pprrooggrraammmmiinngg aallggoorriitthhmm wwiiddeellyy uusseedd ttoo ssoollvvee llaarrggee ssccaallee 
pprroobblleemmss.. AAlltthhoouugghh iitt llaacckkss tthhee iinnttuuiittiivvee aappppeeaall ooff tthhee 
ggrraapphhiiccaall aapppprrooaacchh,, iittss aabbiilliittyy ttoo hhaannddllee pprroobblleemmss wwiitthh 
mmoorree tthhaann ttwwoo ddeecciissiioonn vvaarriiaabblleess mmaakkeess iitt eexxttrreemmeellyy 
vvaalluuaabbllee ffoorr ssoollvviinngg pprroobblleemmss oofftteenn eennccoouunntteerreedd iinn 
pprroodduuccttiioonn//ooppeerraattiioonnss mmaannaaggeemmeenntt.. 
 TThhuuss ssiimmpplleexx mmeetthhoodd ooffffeerrss aann eeffffiicciieenntt mmeeaannss ooff 
ssoollvviinngg mmoorree ccoommpplleexx lliinneeaarr pprrooggrraammmmiinngg pprroobblleemmss.. 
2-67 
67
2-68 
Characteristics of Simplex Method 
 In the simplex method, the computational rroouuttiinnee iiss aann iitteerraattiivvee 
pprroocceessss.. TToo iitteerraattee mmeeaannss ttoo rreeppeeaatt;; hheennccee,, iinn wwoorrkkiinngg ttoowwaarrdd 
tthhee ooppttiimmuumm ssoolluuttiioonn,, tthhee ccoommppuuttaattiioonnaall rroouuttiinnee iiss rreeppeeaatteedd 
oovveerr aanndd oovveerr,, ffoolllloowwiinngg aa ssttaannddaarrdd ppaatttteerrnn.. 
 SSuucccceessssiivvee ssoolluuttiioonnss aarree ddeevveellooppeedd iinn aa ssyysstteemmaattiicc ppaatttteerrnn uunnttiill 
tthhee bbeesstt ssoolluuttiioonn iiss rreeaacchheedd.. 
 EEaacchh nneeww ssoolluuttiioonn wwiillll yyiieelldd aa vvaalluuee ooff tthhee oobbjjeeccttiivvee ffuunnccttiioonn 
aass llaarrggee aass oorr llaarrggeerr tthhaann tthhee pprreevviioouuss ssoolluuttiioonn.. TThhiiss iimmppoorrttaanntt 
ffeeaattuurree aassssuurreess uuss tthhaatt wwee aarree aallwwaayyss mmoovviinngg cclloosseerr ttoo tthhee 
ooppttiimmuumm aannsswweerr.. FFiinnaallllyy,, tthhee mmeetthhoodd iinnddiiccaatteess wwhheenn tthhee 
ooppttiimmuumm ssoolluuttiioonn hhaass bbeeeenn rreeaacchheedd.. 
68
2-69 
Characteristics of Simplex Method 
 lif Most real-lifee lliinneeaarr pprrooggrraammmmiinngg pprroobblleemmss 
hhaavvee mmoorree tthhaann ttwwoo vvaarriiaabblleess,, ssoo aa pprroocceedduurree 
ccaalllleedd tthhee ssiimmpplleexx mmeetthhoodd iiss uusseedd ttoo ssoollvvee ssuucchh 
pprroobblleemmss.. TThhiiss pprroocceedduurree ssoollvveess tthhee pprroobblleemm iinn 
aann iitteerraattiivvee mmaannnneerr,, tthhaatt iiss,, rreeppeeaattiinngg tthhee ssaammee 
sseett ooff pprroocceedduurreess ttiimmee aafftteerr ttiimmee uunnttiill aann 
ooppttiimmaall ssoolluuttiioonn iiss rreeaacchheedd.. EEaacchh iitteerraattiioonn 
bbrriinnggss aa hhiigghheerr vvaalluuee ffoorr tthhee oobbjjeeccttiivvee ffuunnccttiioonn 
ssoo tthhaatt wwee aarree aallwwaayyss mmoovviinngg cclloosseerr ttoo tthhee 
ooppttiimmaall ssoolluuttiioonn.. 
69
Characteristics of Simplex Method 
The simplex method requires simple mathematical 
operations (addition, subtraction, multiplication, and 
division), but the computations are lengthy and tedious, and 
the slightest error can lead to a good deal of frustration. For 
these reasons, most users of the technique rely on computers 
to handle the computations while they concentrate on the 
solutions. Still, some familiarity with manual computations 
is helpful in understanding the simplex process. The student 
will discover that it is better not to use his/her calculator in 
working through these problems because rounding can 
easily distort the results. Instead, it is better to work with 
numbers in fractional form. 
2-70 
70
• It is important to understand the ideas used to produce 
solution. The simplex approach yields not only the optimal 
solution to the xi variables, and the maximum profit (or 
minimum cost) but valuable economic information as well. 
• To be able to use computers successfully and to interpret 
LP computer print outs, we need to know what the simplex 
method is doing and why. 
• We begin by solving a maximization problem using the 
simplex method. We then tackle a minimization problem. 
2-71 
Why we should study the Simplex Method? 
71
Step 1. Formulate a LP model of the problem. 
Step 2. Add slack variables to each constraint to obtain standard form. 
Step 3. Set up the initial simplex tableau. 
Step 4. Choose the nonbasic variable with the largest entry in the net evaluation row 
(Cj – Zj) to bring into the basis. This identifies the pivot (key) column; the 
column associated with the incoming variable. 
Step 5. Choose as the pivot row that row with the smallest ratio of “bi/ aij”, for aij >0 
2-72 
SUMMARY OF THE SIMPLEX METHOD 
where j is the pivot column. This identifies the pivot row, the row of the 
variable leaving the basis when variable j enters. 
Step 6. a). Divide each element of the pivot row by the pivot element. 
b). According to the entering variable, find the new values for remaining 
72 
variables. 
Step 7. Test for optimality. If Cj – Zj £ 0 for all columns, we have the optimal 
solution. If not, return to step 4.
2-73 
73 
Example 1 
A Furniture Ltd., wants to determine the most profitable combination of 
products to manufacture given that its resources are limited. The 
Furniture Ltd., makes two products, tables and chairs, which must be 
processed through assembly and finishing departments. Assembly has 
60 hours available; Finishing can handle up to 48 hours of work. 
Manufacturing one table requires 4 hours in assembly and 2 hours in 
finishing. 
Each chair requires 2 hours in assembly and 4 hours in finishing. Profit 
is $8 per table and $6 per chair.
2-74 
Tabular solution for Example 1/1 
74
2-75 
Tabular solution for Example 1/2 
75
2-76 
Tabular solution for Example 1/3 
76
2-77 
Tabular solution for Example 1/4 
77
2-78 
Tabular solution for Example 1/5 
78
2-79 
Tabular solution for Example 1/6 
79
2-80 
Tabular solution for Example 1/7 
80
2-81 
Tabular solution for Example 1/8 
81
Example 2 
PAR Inc. produces golf equipment and decided to move into the market for standard 
Cutting and dyeing the material, 
Sewing, 
Finishing (inserting umbrella holder, club separators etc.), 
Inspection and packaging. 
Each standard golf-bag will require 7/10 hr. in the cutting and dyeing department, 1/2 
hr. in the sewing department, 1 hr. in the finishing department and 1/10 hr. in the 
inspection & packaging department. 
Deluxe model will require 1 hr. in the cutting and dyeing department, 5/6 hr. for 
The profit contribution for every standard bag is 10 MU and for every deluxe bag is 9 
2-82 
and deluxe golf bags. Each golf bag requires the following operations: 
sewing, 2/3 hr. for finishing and 1/4 hr. for inspection and packaging 
82 
MU. 
In addition the total hours available during the next 3 months are as follows: 
Cutting & dyeing dept 630 hrs 
Sewing dept 600 hrs 
Finishing 708 hrs 
Inspection & packaging 135 hrs 
The company’s problem is to determine how many standard and deluxe bags should be 
produced in the next 3 months?
2-83 
83 
Example 2
2-84 
Tabular solution for Example 2/1 
84
2-85 
Tabular solution for Example 2/2 
85
2-86 
Tabular solution for Example 2/3 
86
2-87 
Tabular solution for Example 2/4 
87
2-88 
Tabular solution for Example 2/5 
88
2-89 
89 
Example 3 
High Tech industries import components for production of two different models of 
personal computers, called deskpro and portable. High Tech’s management is 
currently interested in developing a weekly production schedule for both products. 
The deskpro generates a profit contribution of $50/unit, and portable generates a 
profit contribution of $40/unit. For next week’s production, a max of 150 hours of 
assembly time is available. Each unit of deskpro requires 3 hours of assembly time. 
And each unit of portable requires 5 hours of assembly time. 
High Tech currently has only 20 portable display components in inventory; thus no 
more than 20 units of portable may be assembled. Only 300 sq. feet of warehouse 
space can be made available for new production. Assembly of each Deskpro requires 
8 sq. ft. of warehouse space, and each Portable requires 5 sq. ft. of warehouse space.
2-90 
Tabular solution for Example 3/1 
90
2-91 
Tabular solution for Example 3/2 
91
2-92 
Tabular solution for Example 3/3 
92
2-93 
Tabular solution for Example 3/3 
93
2-94 
Tableau Form : The Special Case 
 Obtaining tableau form is somewhat 
more complex if the LP contains ³ 
constraints, = constraints, and/or “-ve” 
right-hand-side values. Here we will 
explain how to develop tableau form for 
each of these situations. 
94
 Suppose that in the high-tech industries problem, management 
wanted to ensure that the combined total production for both models 
would be at least 25 units. 
 Thus, 
 Objective Function Max Z = 50X1 + 40X2 
 1X1 £ 20 Portable display 
 8X1 + 5 X2 £ 300 Warehouse space 
 1X1 + 1X2 ³ 25 Min. total production 
 X1 , X2 ³ 0 
2-95 
Tabular solution for Example 4/1 
Subjective to : 3 X1 + 5 X2 £ 150 Assembly time 
95
 First, we use three slack variables and one surplus 
variable to write the problem in std. Form. 
 Max Z = 50X1 + 40X2 + 0S1 + 0S2 + 0S3 + 0S4 
 Subject to 3X1 + 5X2 + 1S1 = 150 
 1X2 + 1S2 = 20 
 8X1 + 5X2 + 1S3 = 300 
 1X1 + 1X2 - 1S4 = 25 
 All variables ³ 0 
 For the initial tableau X1 = 0 X2 = 0 
 S1= 150 S2 = 20 
 S3 = 300 S4 = -25 
2-96 
Tabular solution for Example 4/2 
96
 Clearly this is not a basic feasible solution since 
S4 = -25 violates the nonnegativity requirement. 
 We introduce new variable called ARTIFICIAL 
VARIABLE. 
 Artificial variables will be eliminated before the optimal 
solution is reached. We assign a very large cost to the 
variable in the objective function. 
 Objective function 
50X1 + 40X2 + 0S1 + 0S2 + 0S3 + 0S4 - MA4 
2-97 
Tabular solution for Example 4/3 
97
2-98 
Tabular solution for Example 4/4 
Initial Tableau 
Cj 50 40 0 0 0 0 -M 
Max. (Entering) 
98 
Product 
mix 
Quantit 
bi 
X1 X2 S1 S2 S3 S4 A4 bi / aij 
0 S1 150 3 5 1 0 0 0 0 150/3=50 
0 S2 20 0 1 0 1 0 0 0 -- 
0 S3 300 8 5 0 0 1 0 0 300/8=37.5 
-M A4 25 1 1 0 0 0 -1 1 25 
Min. leaving 
Zj -25M -M -M 0 0 0 M -M 
Cj – Zj 50+M 40+M 0 0 0 -M 0 
New X1 values = 25, 1, 1, 0, 0, 0, -1, 1
2-99 
Tabular solution for Example 4/5 
99
2-100 
Tabular solution for Example 4/6 
2nd Tableau 
Cj $50 40 0 0 0 0 -M 
100 
Prodt 
mix 
Quant 
bi 
X1 X2 S1 S2 S3 S4 A4 bi / aij 
$0 S1 75 0 2 1 0 0 3 -3 75/3=25 
0 S2 20 0 1 0 1 0 0 0 -- 
0 S3 100 0 -3 0 0 1 8 -8 100/8=12.5 
Min,leaving 
50 X1 25 1 1 0 0 0 -1 1 -- 
Zj $1250 50 50 0 0 0 -50 50 
Cj – Zj 0 -10 0 0 0 50 -M-50 
Max. (Entering) 
new S4 values : 100/8 = 25/2, 0, -3/8, 0, 0, 1/8, 1
2-101 
Tabular solution for Example 4/7 
 IMPORTANT!! 
 Since A4 is an artificial variable that was 
added simply to obtain an initial basic 
feasible solution, we can drop its associated 
column from the simplex tableau. 
 Indeed whenever artificial variables are 
used, they can be dropped from the 
simplex tableau as soon as they have been 
eliminated from the basic feasible solution. 
101
2-102 
Tabular solution for Example 4/8 
102
2-103 
Tabular solution for Example 4/9 
One more iteration is required. This time X2 comes into the solution and S1 
is eliminated. After performing this iteration, the following simplex tableau 
shows that the optimal solution has been reached. 
103
2-104 
104 
Tabular solution for Example 4/10
EQUALITY CONTRAINTS 
NEGATIVE RIGHT-HAND SIDE VALUES 
 Simply add an artificial variable A1 to create a basic feasible 
solution in the initial simplex tableau. 
6X1 + 4X2 - 5X3 = 30 Þ6X1 + 4X2 - 5X3 + 1A1 = 30 
 One of the properties of the tableau form of a linear 
program is that the values on the right-hand sides of the 
constraints have to be nonnegative. 
 e.g. # of units of the portable model (X2) has to be less 
than or equal to the # of units of the deskpro model (X1) 
after setting aside 5 units of the deskpro for internal 
company use. 
2-105 
105 
X2 £ X1 - 5 
 - X1 + X2 £ -5 
 (Min)Multiply by –1 Þ (Max) X1 - X2 ³ 5 
 We now have an acceptable nonnegative right-hand-side value. 
Tableau form for this constraint can now be obtained by subtracting a 
surplus variable and adding an artificial variable.
 Livestock Nutrition Co. produces specially blended feed 
supplements. LNC currently has an order for 200 kgs of 
its mixture. 
 This consists of two ingredients 
X1 ( a protein source ) 
X2 ( a carbohydrate source ) 
 The first ingredient, X1 costs LNC 3MU a kg. The second 
ingredient, X2 costs LNC 8MU a kg. The mixture can’t be 
more than 40% X1 and it must be at least 30% X2. 
 LNC’s problem is to determine how much of each 
ingredient to use to minimize cost. 
2-106 
Tabular solution for Example 5/1 
106
2-107 
Tabular solution for Example 5/2 
 The cost function can be written as 
Cost = 3X1 + 8X2 Min! 
 LNC must produce 200 kgs of the mixture – no more, no 
less. 
X1 + X2 = 200 kgs 
 The mixture can’t be more than 40% X1, so we may use 
less than 80 kgs. (40% X 200 = 80). However, we must 
not exceed 80 kgs. 
X1 £ 80 kgs 
 The mixture must be at least 30% X2. Thus we may use 
more than 60 kgs, not less than 60 kgs. (30% X 200 = 
60) 
 X2 ³ 60 kgs 
107
2-108 
Tabular solution for Example 5/3 
Minimize : Cost = 3MU X1 + 8MU X2 
Subject to X1 + X2 = 200 kgs 
X1 £ 80 kgs 
X2 ³ 60 kgs 
X1 , X2 ³ 0 
 An initial solution: X1 + X2 = 200 kgs 
Þ X1 + X2 + A1 = 200 
108 
ß 
 Artificial variable : A very expensive substance must not 
be represented in optimal solution.
2-109 
Tabular solution for Example 5/4 
 An artificial Variable is only of value as a computational device; it 
allows 2 types of restrictions to be treated. 
 The equality type 
 ³ type 
X1 £ 80 kgs constraint on protein 
Þ X1 + S1 = 80 kgs 
X2 ³ 60 kgs constraint on carbohydrates 
Þ X2 - S2 + A2 = 60 
 X1 , X2 , S1, S2, A1, A2 ³ 0 
ß ß ß ß 
0MU 0MU M M 
109
2-110 
Tabular solution for Example 5/5 
Minimize : Cost = 3X1 + 8X2 + 0S1 + 0S2 + MA1 + MA2 
Subject to : X1 + X2 + A1 = 
200 
X1 + S1 = 80 
X2 - S2 + A2 = 60 
110 
All variables ³ 0
2-111 
Tabular solution for Example 5/6 
111
2-112 
Tabular solution for Example 5/7 
112
2-113 
Tabular solution for Example 5/8 
113
2-114 
Tabular solution for Example 5/9 
114
2-115 
Tabular solution for Example 5/10 
115
2-116 
Tabular solution for Example 5/11 
116
2-117 
Tabular solution for Example 5/12 
117
Copyright © 2010 Pearson Education, Inc. Publishing as Prentice Hall 2-118
Copyright © 2010 Pearson Education, Inc. Publishing as Prentice Hall 2-119
B 
A 
C 
D 
Copyright © 2010 Pearson Education, Inc. Publishing as Prentice Hall 2-120
Copyright © 2010 Pearson Education, Inc. Publishing as Prentice Hall 2-121
Copyright © 2010 Pearson Education, Inc. Publishing as Prentice Hall 2-122
Copyright © 2010 Pearson Education, Inc. Publishing as Prentice Hall 2-123
2-124 
Special cases in simplex method 
 Degeneracy 
 Alternate optima 
 Unbounded solution 
 Nonexisting sol(infeasible) 
Copyright © 2010 Pearson Education, Inc. Publishing as Prentice Hall
2-125 
Degeneracy 
 At least one basic variable is zero 
 It occurs when there is a tie for minimum ratio 
 At least one basic variable will be zero in the next iteration 
 Degeneracy can cause to cycle indefinitely 
 This reveals at least one redundant constraint 
Copyright © 2010 Pearson Education, Inc. Publishing as Prentice Hall
2-126 
Example 
 Max z = 3x + 9 y 
 S.t. X + 4 y £ 8 
 x + 2 y £ 4 
 x, y ³ 0 
 Solve by simplex and check 
Copyright © 2010 Pearson Education, Inc. Publishing as Prentice Hall
Figure 3.7 LP degeneracy in 
Example 3.5-1 
Copyright © 2010 Pearson Education, Inc. Publishing as Prentice Hall 2-127
2-128 
Alternate optima 
 zero coef of non basic x indicate that x 
can be made basic,altering the values of the 
basic variables without changing the value 
of z. 
 max z = 2x + 4 y 
 S.t x + 2 y £ 5 
 x + y £ 4 
 x, y ³ 0 
Copyright © 2010 Pearson Education, Inc. Publishing as Prentice Hall
Figure 3.9 LP alternative optima 
in Example 3.5-2 
Copyright © 2010 Pearson Education, Inc. Publishing as Prentice Hall 2-129
2-130 
Unbounded solution 
 if in the key col all the coeff are non 
positive 
 New var can be increased indefinitely 
without violating any of the constraints 
 Hence z can be increases indefinitely 
Copyright © 2010 Pearson Education, Inc. Publishing as Prentice Hall
2-131 
example 
 Max z = 2x + y 
 S.t 
x – y £10 
2x £ 40 
x, y ³ 0 
Copyright © 2010 Pearson Education, Inc. Publishing as Prentice Hall
Figure 3.10 LP unbounded 
solution in Example 3.5-3 
Copyright © 2010 Pearson Education, Inc. Publishing as Prentice Hall 2-132
2-133 
Infeasible sol 
Case does not occur if all constraints are less 
than equal to 
If at least one artificial variable is positive in 
the optimum iteration the LP has no feasible 
sol 
Copyright © 2010 Pearson Education, Inc. Publishing as Prentice Hall
2-134 
example 
 Max z = 3x + 2 y 
 2x + y £2 
 3x + 4 y £12 
 x, y³ 0 
 Check by solving 
Copyright © 2010 Pearson Education, Inc. Publishing as Prentice Hall
Figure 3.11 Infeasible solution of 
Example 3.5-4 
Copyright © 2010 Pearson Education, Inc. Publishing as Prentice Hall 2-135
2-136 
Duality Theory 
The theory of duality is a very 
elegant and important concept 
within the field of operations 
research.
2-137 
The notion of duality within linear 
programming asserts that every linear 
program has associated with it a related 
linear program called its dual. The original 
problem in relation to its dual is termed 
the primal.
2-138 
Examples 
There is a small company in Melbourne which has 
recently become engaged in the production of office 
furniture. 
The company manufactures tables, desks and chairs. The 
production of a table requires 8 kgs of wood and 5 kgs 
of metal and is sold for $80; a desk uses 6 kgs of wood 
and 4 kgs of metal and is sold for $60; and a chair requires 
4 kgs of both metal and wood and is sold for $50. 
We would like to determine the revenue maximizing 
strategy for this company, given that their resources are 
limited to 100 kgs of wood and 60 kgs of metal.
2-139 
Problem P1 
Z = 80x1 + 60x2 + 50x3 
8 6 4 100 
5 4 4 60 
max 
x 
x x x 
x x x 
x x x 
+ + £ 
+ + £ 
, , ³ 
1 2 3 
1 2 3 
1 2 3 
0
2-140 
Now consider that there is a much bigger 
company in Melbourne which has been 
the lone producer of this type of furniture 
for many years. They don't appreciate 
the competition from this new company; 
so they have decided to tender an offer to 
buy all of their competitor's resources and 
therefore put them out of business.
The challenge for this large company then is 
to develop a linear program which will 
determine the appropriate amount of 
money that should be offered for a unit of 
each type of resource, such that the offer will 
be acceptable to the smaller company while 
minimizing the expenditures of the larger 
company. 
2-141
2-142 
Problem D1 
y y 
y y 
y y 
y y 
+ ³ 
+ ³ 
+ ³ 
, ³ 
8 5 80 
6 4 60 
4 4 50 
0 
1 2 
1 2 
1 2 
1 2 
min 
y 
w = 100y1 + 60y2
2-143 
A Diet Problem 
An individual has a choice of two types of food to eat, 
meat and potatoes, each offering varying degrees of 
nutritional benefit. He has been warned by his doctor that 
he must receive at least 400 units of protein, 200 units of 
carbohydrates and 100 units of fat from his daily diet. 
Given that a kg of steak costs $10 and provides 80 units of 
protein, 20 units of carbohydrates and 30 units of fat, and 
that a kg of potatoes costs $2 and provides 40 units of 
protein, 50 units of carbohydrates and 20 units of fat, he 
would like to find the minimum cost diet which satisfies his 
nutritional requirements
2-144 
Problem P2 
80 40 400 
20 50 200 
30 20 100 
1 2 
1 2 
1 2 
1 2 
0 
x x 
x x 
x x 
x x 
+ ³ 
+ ³ 
+ ³ 
, ³ 
min 
x 
Z =10x1 + 2x2
2-145 
Now consider a chemical company which 
hopes to attract this individual away from 
his present diet by offering him synthetic 
nutrients in the form of pills. This 
company would like determine prices per 
unit for their synthetic nutrients which will 
bring them the highest possible revenue 
while still providing an acceptable dietary 
alternative to the individual.
2-146 
Problem D2 
max 
y 
w = 400y1 + 200y2 + 100y3 
80y1 + 20y2 + 30y3 £10 
40y1 + 50y2 + 20y3 £ 2 
y1, y2, y3 ³ 0
2-147 
Comments 
Each of the two examples describes some 
kind of competition between two 
decision makers. 
We shall investigate the economic 
interpretation of the primal/dual 
relationship later in this chapter.
2-148 
FINDING THE DUAL 
OF A STANDARD 
LINEAR PROGRAM 
In this section we formalize the intuitive 
feelings we have with regard to the 
relationship between the primal and dual 
versions of the two illustrative examples 
we examined 
The important thing to observe is that the 
relationship - for the standard form - is 
given as a definition.
2-149 
Standard form of the Primal 
Problem 
a x + a x + ... 
+ a x £ 
b 
a x + a x + ... 
+ a x £ 
b 
11 1 12 2 1 1 
21 1 22 2 2 2 
... ... ... ... 
... ... ... ... 
a x + a x + ... 
+ a x £ 
b 
x x x 
n n 
n n 
m m mn n m 
n 
1 1 2 2 
³ 
, ,..., 
0 
1 2 max 
x 
j j 
j 
n 
Z = å c x 
=1
2-150 
Standard form of the Dual 
Problem 
min 
y 
a y + a y + ... 
+ a y ³ 
c 
a y + a y + ... 
+ a y ³ 
c 
11 1 21 2 1 1 
12 1 22 2 2 2 
... ... ... ... 
... ... ... ... 
a y + a y + ... 
+ a y ³ 
c 
y y y 
m m 
m m 
n n mn m n 
m 
1 1 2 2 
³ 
, ,..., 
0 
1 2 i 
i 
m 
i w b y = å= 
1
2-151 
Definition 
Primal Problem Dual Problem 
z *: = max 
Z = 
cx 
x 
s . t 
. 
Ax £ 
b 
x 
³ 0 
=min 
w*:x 
w=yb 
s.t. 
yA³c 
y ³0 
b is not assumed to be non-negative
2-152 
Example 
Primal 
x x x x 
x x x x x 
- + - £ 
+ - + + £ 
3 8 9 15 20 
18 5 8 4 12 30 
1 2 4 5 
1 2 3 4 5 
0 
x x x x x 
, , , , ³ 
1 2 3 4 5 
max 
x 
Z = 5x1 + 3x2 - 8x3 + 0x4 +12x5
2-153 
w = 20y1 + 30y2 
3 18 5 
8 5 3 
min 
y 
+ ³ 
1 2 
1 2 
- + ³ 
- ³ - 
+ ³ 
8 8 
2 
9 4 0 
15 12 12 
1 2 
1 2 
1 2 
0 
y y 
y y 
y 
y y 
y y 
y y 
- + ³ 
, ³ 
Dual
2-154 
Primal-Dual relationship 
x1 ³ 0 x2 ³ 0 xn ³ 0 w= 
y1 ³ 0 a11 a12 a1n £ b1 
Dual y2 ³ 0 a21 a22 a2n £ b2 
(min w) ... ... ... ... ... 
ym ³ 0 am1 am2 amn £ bn 
³ ³ ³ 
Z= c1 c2 cn
2-155 
Example 
x x x 
x x x 
x x x 
x x 
- + £ 
5 18 5 15 
- 8 + 12 - 8 £ 
8 
12 - 4 + 8 £ 
10 
2 5 5 
- £ 
, , ³ 
0 
1 2 3 
1 2 3 
1 2 3 
1 3 
x x x 
1 2 3 
max 
x 
Z = 4x1 +10x2 - 9x3
2-156 
Conversion 
Multiply through the greater-than-or-equal-to 
inequality constraint by -1 
Use the approach described above to 
convert the equality constraint to a pair of 
inequality constraints. 
Replace the variable unrestricted in sign, , by 
the difference of two nonnegative variables.
2-158 
Dual 
y2 - y3 + y4 ³ 1 
-2y1 - 3y2 + 3y3 - 2y4 ³ 1 
y1 + 4y2 - 4y3 ³ 1 
- y1 - 4y2 + 4y3 ³ -1 
y1, y2, y3, y4 ³ 0 
min 
y 
w = -4y1 + 5y2 - 5y3 + 3y4
2-159 
Streamlining the 
conversion ... 
An equality constraint in the primal 
generates a dual variable that is 
unrestricted in sign. 
An unrestricted in sign variable in the 
primal generates an equality constraint 
in the dual.
2-160 
(Continued) 
min 
y 
w = -4y1 + 5y2 - 5y3 + 3y4 
y2 - y3 + y4 ³ 1 
-2y1 - 3y2 + 3y3 - 2y4 ³ 1 
y1 + 4y2 - 4y3 ³ 1 
- y1 - 4y2 + 4y3 ³ -1 
y1, y2, y3, y4 ³ 0
2-161 
 + w = -4y1 + 5y2 + 3y3 
, , 
y y 
+ ³ 
2 3 
, , , 
, , 
, , , ; , 
y y y 
y y 
y y y 
- - - ³ 
1 2 3 
1 2 
1 3 2 
1 
2 3 2 1 
- + = 
4 1 
³ 0 
urs 
min 
, 
, , , 
y 
+ 
correction
2-162 
Primal-Dual relationship 
Primal Problem 
opt=max 
Constraint i : 
= form 
= form 
Variable j: 
xj = 0 
xj urs 
Dual Problem 
opt=min 
Variable i : 
yi = 0 
yi urs 
Constraint j: 
= form 
= form
2-163 
Example 
x x 
x x 
x 
x x 
- ³ - 
+ = - 
3 8 6 
1 2 
1 2 
1 
2 1 
6 5 
8 10 
0 
= 
³ ; urs 
max 
x 
Z = 5x1 + 4x2
2-164 
equivalent non-standard 
form 
max 
x 
Z = 5x1 + 4x2 
- x + x 
£ 
x x 
x 
x ; x urs 
3 8 6 
1 2 
1 2 
+ = - 
= 
³ 
6 5 
8 10 
0 
1 
2 1
2-165 
Dual from the recipe 
y y y 
y y 
y ; y , y urs 
- + + = 
3 8 5 
8 6 4 
1 2 3 
1 2 
1 2 3 
+ ³ 
³ 
0 
min 
y 
w = 6y1 - 5y2 +10y3
2-166 
What about opt=min ? 
Can use the usual trick of multiplying the 
objective function by -1 (remembering to 
undo this when the dual is constructed.) 
It is instructive to use this method to 
construct the dual of the dual of the 
standard form. 
i.e, what is the dual of the dual of the 
standard primal problem?
2-167 
What is the dual of 
=min 
w*:x 
w=yb 
s.t. 
yA³c 
y³0
2-168 
max 
y 
- w = - yb 
s.t. 
yA ³ c 
y ³ 0 
max 
y 
- w = -yb 
s.t. 
- yA £ -c 
y ³ 0
2-169 
min 
x 
. . 
Z cx 
s t 
- = - 
Ax b 
x 
- ³ - 
³ 0 
max 
x 
. . 
Z cx 
s t 
= 
Ax £ 
b 
x 
³ 0
2-170 
Important Observation 
FOR ANY PRIMAL LINEAR 
PROGRAM, THE DUAL OF THE 
DUAL IS THE PRIMAL
2-171 
Primal-Dual Relationship 
Primal or Dual 
opt=max opt=min 
Dual or Primal 
Variable i : 
yi = 0 
yi urs 
Constraint j: 
= form 
= form 
Constraint i : 
= form 
= form 
Variable j: 
xj = 0 
xj urs
2-172 
Example 
min 
x 
Z = 6x1 + 4x2 
x x 
x x 
x x 
x x 
- ³ 
+ = - 
- £ 
, ³ 
3 5 12 
2 8 
5 10 
0 
1 2 
1 2 
1 2 
1 1
2-173 
equivalent form 
min 
x 
Z = 6x1 + 4x2 
3 5 12 
1 2 
1 2 
2 8 
5 10 
1 2 
1 2 
0 
x x 
x x 
x x 
x x 
- ³ 
+ = - 
- + ³ - 
, ³
2-174 
Dual 
y y y 
y y y 
y y y 
+ - £ 
3 5 6 
5 2 4 
1 2 3 
1 2 3 
1 3 2 
- + + £ 
, ³ 0 
; urs 
max 
y 
w =12y1 - 8y2 - 10y3
2-175 
Exercise 
Maximize z = 3 x + 9 y 
such that x + 4 y ≤ 8 
x + 2 y ≤4 
x, y≥0 
Determine its dual ,Solve both the problems 
and compare the results

More Related Content

What's hot

Assembly Line Balancing -Example
Assembly Line Balancing -ExampleAssembly Line Balancing -Example
Assembly Line Balancing -ExampleJoseph Konnully
 
Mba i qt unit-1.3_linear programming in om
Mba i qt unit-1.3_linear programming in omMba i qt unit-1.3_linear programming in om
Mba i qt unit-1.3_linear programming in omRai University
 
Unit.3. duality and sensetivity analisis
Unit.3. duality and sensetivity analisisUnit.3. duality and sensetivity analisis
Unit.3. duality and sensetivity analisisDagnaygebawGoshme
 
Strategic capacity planning for products and services
Strategic capacity planning for products and servicesStrategic capacity planning for products and services
Strategic capacity planning for products and servicesgerlyn bonus
 
Chapter 10 human_resources_and_job_design
Chapter 10 human_resources_and_job_designChapter 10 human_resources_and_job_design
Chapter 10 human_resources_and_job_designNên Trần Ngọc
 
Linear Programming Application
Linear Programming ApplicationLinear Programming Application
Linear Programming ApplicationKashif Latif
 
Simplex method
Simplex methodSimplex method
Simplex methodtatteya
 
Linear Programming Problems : Dr. Purnima Pandit
Linear Programming Problems : Dr. Purnima PanditLinear Programming Problems : Dr. Purnima Pandit
Linear Programming Problems : Dr. Purnima PanditPurnima Pandit
 
Chapter 8 ror analysis for multiple alternatives
Chapter 8   ror analysis for multiple alternativesChapter 8   ror analysis for multiple alternatives
Chapter 8 ror analysis for multiple alternativesBich Lien Pham
 
Chapter 1 introduction to engineering economy
Chapter 1 introduction to engineering economy Chapter 1 introduction to engineering economy
Chapter 1 introduction to engineering economy Fhatiha Atika
 
Integer Programming, Goal Programming, and Nonlinear Programming
Integer Programming, Goal Programming, and Nonlinear ProgrammingInteger Programming, Goal Programming, and Nonlinear Programming
Integer Programming, Goal Programming, and Nonlinear ProgrammingSalah A. Skaik - MBA-PMP®
 
Quantity discount
Quantity discountQuantity discount
Quantity discountHriday Bora
 
The Big M Method - Operation Research
The Big M Method - Operation ResearchThe Big M Method - Operation Research
The Big M Method - Operation Research2013901097
 
Quiz 1 Engineering Economy
Quiz 1 Engineering EconomyQuiz 1 Engineering Economy
Quiz 1 Engineering EconomyAbel Nyarangi
 

What's hot (20)

Chapter 3-2
Chapter 3-2Chapter 3-2
Chapter 3-2
 
Assembly Line Balancing -Example
Assembly Line Balancing -ExampleAssembly Line Balancing -Example
Assembly Line Balancing -Example
 
Mba i qt unit-1.3_linear programming in om
Mba i qt unit-1.3_linear programming in omMba i qt unit-1.3_linear programming in om
Mba i qt unit-1.3_linear programming in om
 
Unit.3. duality and sensetivity analisis
Unit.3. duality and sensetivity analisisUnit.3. duality and sensetivity analisis
Unit.3. duality and sensetivity analisis
 
Strategic capacity planning for products and services
Strategic capacity planning for products and servicesStrategic capacity planning for products and services
Strategic capacity planning for products and services
 
Goal programming
Goal programmingGoal programming
Goal programming
 
Chapter 10 human_resources_and_job_design
Chapter 10 human_resources_and_job_designChapter 10 human_resources_and_job_design
Chapter 10 human_resources_and_job_design
 
Linear Programming Application
Linear Programming ApplicationLinear Programming Application
Linear Programming Application
 
Simplex method
Simplex methodSimplex method
Simplex method
 
Linear Programming Problems : Dr. Purnima Pandit
Linear Programming Problems : Dr. Purnima PanditLinear Programming Problems : Dr. Purnima Pandit
Linear Programming Problems : Dr. Purnima Pandit
 
Simplex Algorithm
Simplex AlgorithmSimplex Algorithm
Simplex Algorithm
 
Chapter 8 ror analysis for multiple alternatives
Chapter 8   ror analysis for multiple alternativesChapter 8   ror analysis for multiple alternatives
Chapter 8 ror analysis for multiple alternatives
 
Chapter 1 introduction to engineering economy
Chapter 1 introduction to engineering economy Chapter 1 introduction to engineering economy
Chapter 1 introduction to engineering economy
 
5. transportation problems
5. transportation problems5. transportation problems
5. transportation problems
 
Integer Programming, Goal Programming, and Nonlinear Programming
Integer Programming, Goal Programming, and Nonlinear ProgrammingInteger Programming, Goal Programming, and Nonlinear Programming
Integer Programming, Goal Programming, and Nonlinear Programming
 
Quantity discount
Quantity discountQuantity discount
Quantity discount
 
The Big M Method - Operation Research
The Big M Method - Operation ResearchThe Big M Method - Operation Research
The Big M Method - Operation Research
 
Operations Research - Introduction
Operations Research - IntroductionOperations Research - Introduction
Operations Research - Introduction
 
Rsh qam11 ch09 ge
Rsh qam11 ch09 geRsh qam11 ch09 ge
Rsh qam11 ch09 ge
 
Quiz 1 Engineering Economy
Quiz 1 Engineering EconomyQuiz 1 Engineering Economy
Quiz 1 Engineering Economy
 

Viewers also liked

Operations research - an overview
Operations research -  an overviewOperations research -  an overview
Operations research - an overviewJoseph Konnully
 
Simplextabular
SimplextabularSimplextabular
Simplextabularhushgas
 
Operations Research
Operations ResearchOperations Research
Operations Researchajithsrc
 
Operational research in Public Health in India
Operational research in Public Health in IndiaOperational research in Public Health in India
Operational research in Public Health in IndiaDr. Dharmendra Gahwai
 
Operation research history and overview application limitation
Operation research history and overview application limitationOperation research history and overview application limitation
Operation research history and overview application limitationBalaji P
 
Numerical analysis simplex method 2
Numerical analysis  simplex method 2Numerical analysis  simplex method 2
Numerical analysis simplex method 2SHAMJITH KM
 
Chapter 1 operations research (2)
Chapter   1 operations research (2)Chapter   1 operations research (2)
Chapter 1 operations research (2)Ashna Singh
 
Solving linear programming model by simplex method
Solving linear programming model by simplex methodSolving linear programming model by simplex method
Solving linear programming model by simplex methodRoshan Kumar Patel
 
Numerical analysis dual, primal, revised simplex
Numerical analysis  dual, primal, revised simplexNumerical analysis  dual, primal, revised simplex
Numerical analysis dual, primal, revised simplexSHAMJITH KM
 
Linear programming using the simplex method
Linear programming using the simplex methodLinear programming using the simplex method
Linear programming using the simplex methodShivek Khurana
 
Simplex method - Maximisation Case
Simplex method - Maximisation CaseSimplex method - Maximisation Case
Simplex method - Maximisation CaseJoseph Konnully
 
Operation Research (Simplex Method)
Operation Research (Simplex Method)Operation Research (Simplex Method)
Operation Research (Simplex Method)Shivani Gautam
 

Viewers also liked (20)

Operations research
Operations researchOperations research
Operations research
 
Operations research - an overview
Operations research -  an overviewOperations research -  an overview
Operations research - an overview
 
Operation research
Operation research Operation research
Operation research
 
Simplextabular
SimplextabularSimplextabular
Simplextabular
 
Operations Research
Operations ResearchOperations Research
Operations Research
 
Operational research in Public Health in India
Operational research in Public Health in IndiaOperational research in Public Health in India
Operational research in Public Health in India
 
Operation research history and overview application limitation
Operation research history and overview application limitationOperation research history and overview application limitation
Operation research history and overview application limitation
 
Lp simplex 3_
Lp simplex 3_Lp simplex 3_
Lp simplex 3_
 
Simplex
SimplexSimplex
Simplex
 
5. advance topics in lp
5. advance topics in lp5. advance topics in lp
5. advance topics in lp
 
Numerical analysis simplex method 2
Numerical analysis  simplex method 2Numerical analysis  simplex method 2
Numerical analysis simplex method 2
 
Chapter 1 operations research (2)
Chapter   1 operations research (2)Chapter   1 operations research (2)
Chapter 1 operations research (2)
 
11-Management Science
11-Management Science11-Management Science
11-Management Science
 
Solving linear programming model by simplex method
Solving linear programming model by simplex methodSolving linear programming model by simplex method
Solving linear programming model by simplex method
 
Linear programming
Linear programmingLinear programming
Linear programming
 
Numerical analysis dual, primal, revised simplex
Numerical analysis  dual, primal, revised simplexNumerical analysis  dual, primal, revised simplex
Numerical analysis dual, primal, revised simplex
 
Linear programming using the simplex method
Linear programming using the simplex methodLinear programming using the simplex method
Linear programming using the simplex method
 
Simplex method - Maximisation Case
Simplex method - Maximisation CaseSimplex method - Maximisation Case
Simplex method - Maximisation Case
 
Operation Research (Simplex Method)
Operation Research (Simplex Method)Operation Research (Simplex Method)
Operation Research (Simplex Method)
 
L20 Simplex Method
L20 Simplex MethodL20 Simplex Method
L20 Simplex Method
 

Similar to Vcs slides on or 2014

LinearProgramming-Graphicalnethod.ppt
LinearProgramming-Graphicalnethod.pptLinearProgramming-Graphicalnethod.ppt
LinearProgramming-Graphicalnethod.pptMayurkumarpatil1
 
1 resource optimization 2
1 resource optimization 21 resource optimization 2
1 resource optimization 2shushay hailu
 
chapter 2 revised.pptx
chapter 2 revised.pptxchapter 2 revised.pptx
chapter 2 revised.pptxDejeneDay
 
chapter 2 revised.pptx
chapter 2 revised.pptxchapter 2 revised.pptx
chapter 2 revised.pptxDejeneDay
 
Linear programming
Linear programmingLinear programming
Linear programmingKrantee More
 
OR chapter 2.pdf
OR chapter 2.pdfOR chapter 2.pdf
OR chapter 2.pdfAlexHayme
 
chapter 2 revised.pptx
chapter 2 revised.pptxchapter 2 revised.pptx
chapter 2 revised.pptxDejeneDay
 
Limitations of linear programming
Limitations of linear programmingLimitations of linear programming
Limitations of linear programmingTarun Gehlot
 
Quantitativetechniqueformanagerialdecisionlinearprogramming 090725035417-phpa...
Quantitativetechniqueformanagerialdecisionlinearprogramming 090725035417-phpa...Quantitativetechniqueformanagerialdecisionlinearprogramming 090725035417-phpa...
Quantitativetechniqueformanagerialdecisionlinearprogramming 090725035417-phpa...kongara
 
LINEAR PROGRAMMING
LINEAR PROGRAMMINGLINEAR PROGRAMMING
LINEAR PROGRAMMINGrashi9
 
Biosight: Quantitative Methods for Policy Analysis - Introduction to GAMS, Li...
Biosight: Quantitative Methods for Policy Analysis - Introduction to GAMS, Li...Biosight: Quantitative Methods for Policy Analysis - Introduction to GAMS, Li...
Biosight: Quantitative Methods for Policy Analysis - Introduction to GAMS, Li...IFPRI-EPTD
 
linearprogramingproblemlpp-180729145239.pptx
linearprogramingproblemlpp-180729145239.pptxlinearprogramingproblemlpp-180729145239.pptx
linearprogramingproblemlpp-180729145239.pptxKOUSHIkPIPPLE
 
OR-I_Lecture_Note_01.pptx
OR-I_Lecture_Note_01.pptxOR-I_Lecture_Note_01.pptx
OR-I_Lecture_Note_01.pptxssuserf19f3e
 
Linear programming manzoor nabi
Linear programming  manzoor nabiLinear programming  manzoor nabi
Linear programming manzoor nabiManzoor Wani
 

Similar to Vcs slides on or 2014 (20)

LinearProgramming-Graphicalnethod.ppt
LinearProgramming-Graphicalnethod.pptLinearProgramming-Graphicalnethod.ppt
LinearProgramming-Graphicalnethod.ppt
 
Ms(lpgraphicalsoln.)[1]
Ms(lpgraphicalsoln.)[1]Ms(lpgraphicalsoln.)[1]
Ms(lpgraphicalsoln.)[1]
 
linearprogramming.pdf
linearprogramming.pdflinearprogramming.pdf
linearprogramming.pdf
 
1 resource optimization 2
1 resource optimization 21 resource optimization 2
1 resource optimization 2
 
chapter 2 revised.pptx
chapter 2 revised.pptxchapter 2 revised.pptx
chapter 2 revised.pptx
 
chapter 2 revised.pptx
chapter 2 revised.pptxchapter 2 revised.pptx
chapter 2 revised.pptx
 
Linear programming
Linear programmingLinear programming
Linear programming
 
OR chapter 2.pdf
OR chapter 2.pdfOR chapter 2.pdf
OR chapter 2.pdf
 
chapter 2 revised.pptx
chapter 2 revised.pptxchapter 2 revised.pptx
chapter 2 revised.pptx
 
Linear Programming
Linear ProgrammingLinear Programming
Linear Programming
 
Limitations of linear programming
Limitations of linear programmingLimitations of linear programming
Limitations of linear programming
 
Quantitativetechniqueformanagerialdecisionlinearprogramming 090725035417-phpa...
Quantitativetechniqueformanagerialdecisionlinearprogramming 090725035417-phpa...Quantitativetechniqueformanagerialdecisionlinearprogramming 090725035417-phpa...
Quantitativetechniqueformanagerialdecisionlinearprogramming 090725035417-phpa...
 
LINEAR PROGRAMMING
LINEAR PROGRAMMINGLINEAR PROGRAMMING
LINEAR PROGRAMMING
 
LPP.pptx
LPP.pptxLPP.pptx
LPP.pptx
 
Biosight: Quantitative Methods for Policy Analysis - Introduction to GAMS, Li...
Biosight: Quantitative Methods for Policy Analysis - Introduction to GAMS, Li...Biosight: Quantitative Methods for Policy Analysis - Introduction to GAMS, Li...
Biosight: Quantitative Methods for Policy Analysis - Introduction to GAMS, Li...
 
Linear programming
Linear programmingLinear programming
Linear programming
 
linearprogramingproblemlpp-180729145239.pptx
linearprogramingproblemlpp-180729145239.pptxlinearprogramingproblemlpp-180729145239.pptx
linearprogramingproblemlpp-180729145239.pptx
 
OR-I_Lecture_Note_01.pptx
OR-I_Lecture_Note_01.pptxOR-I_Lecture_Note_01.pptx
OR-I_Lecture_Note_01.pptx
 
linear programming
linear programminglinear programming
linear programming
 
Linear programming manzoor nabi
Linear programming  manzoor nabiLinear programming  manzoor nabi
Linear programming manzoor nabi
 

Recently uploaded

Application of Residue Theorem to evaluate real integrations.pptx
Application of Residue Theorem to evaluate real integrations.pptxApplication of Residue Theorem to evaluate real integrations.pptx
Application of Residue Theorem to evaluate real integrations.pptx959SahilShah
 
Software and Systems Engineering Standards: Verification and Validation of Sy...
Software and Systems Engineering Standards: Verification and Validation of Sy...Software and Systems Engineering Standards: Verification and Validation of Sy...
Software and Systems Engineering Standards: Verification and Validation of Sy...VICTOR MAESTRE RAMIREZ
 
SPICE PARK APR2024 ( 6,793 SPICE Models )
SPICE PARK APR2024 ( 6,793 SPICE Models )SPICE PARK APR2024 ( 6,793 SPICE Models )
SPICE PARK APR2024 ( 6,793 SPICE Models )Tsuyoshi Horigome
 
HARMONY IN THE HUMAN BEING - Unit-II UHV-2
HARMONY IN THE HUMAN BEING - Unit-II UHV-2HARMONY IN THE HUMAN BEING - Unit-II UHV-2
HARMONY IN THE HUMAN BEING - Unit-II UHV-2RajaP95
 
Model Call Girl in Narela Delhi reach out to us at 🔝8264348440🔝
Model Call Girl in Narela Delhi reach out to us at 🔝8264348440🔝Model Call Girl in Narela Delhi reach out to us at 🔝8264348440🔝
Model Call Girl in Narela Delhi reach out to us at 🔝8264348440🔝soniya singh
 
Biology for Computer Engineers Course Handout.pptx
Biology for Computer Engineers Course Handout.pptxBiology for Computer Engineers Course Handout.pptx
Biology for Computer Engineers Course Handout.pptxDeepakSakkari2
 
Gurgaon ✡️9711147426✨Call In girls Gurgaon Sector 51 escort service
Gurgaon ✡️9711147426✨Call In girls Gurgaon Sector 51 escort serviceGurgaon ✡️9711147426✨Call In girls Gurgaon Sector 51 escort service
Gurgaon ✡️9711147426✨Call In girls Gurgaon Sector 51 escort servicejennyeacort
 
GDSC ASEB Gen AI study jams presentation
GDSC ASEB Gen AI study jams presentationGDSC ASEB Gen AI study jams presentation
GDSC ASEB Gen AI study jams presentationGDSCAESB
 
Call Girls Delhi {Jodhpur} 9711199012 high profile service
Call Girls Delhi {Jodhpur} 9711199012 high profile serviceCall Girls Delhi {Jodhpur} 9711199012 high profile service
Call Girls Delhi {Jodhpur} 9711199012 high profile servicerehmti665
 
HARMONY IN THE NATURE AND EXISTENCE - Unit-IV
HARMONY IN THE NATURE AND EXISTENCE - Unit-IVHARMONY IN THE NATURE AND EXISTENCE - Unit-IV
HARMONY IN THE NATURE AND EXISTENCE - Unit-IVRajaP95
 
VICTOR MAESTRE RAMIREZ - Planetary Defender on NASA's Double Asteroid Redirec...
VICTOR MAESTRE RAMIREZ - Planetary Defender on NASA's Double Asteroid Redirec...VICTOR MAESTRE RAMIREZ - Planetary Defender on NASA's Double Asteroid Redirec...
VICTOR MAESTRE RAMIREZ - Planetary Defender on NASA's Double Asteroid Redirec...VICTOR MAESTRE RAMIREZ
 
microprocessor 8085 and its interfacing
microprocessor 8085  and its interfacingmicroprocessor 8085  and its interfacing
microprocessor 8085 and its interfacingjaychoudhary37
 
Gfe Mayur Vihar Call Girls Service WhatsApp -> 9999965857 Available 24x7 ^ De...
Gfe Mayur Vihar Call Girls Service WhatsApp -> 9999965857 Available 24x7 ^ De...Gfe Mayur Vihar Call Girls Service WhatsApp -> 9999965857 Available 24x7 ^ De...
Gfe Mayur Vihar Call Girls Service WhatsApp -> 9999965857 Available 24x7 ^ De...srsj9000
 
main PPT.pptx of girls hostel security using rfid
main PPT.pptx of girls hostel security using rfidmain PPT.pptx of girls hostel security using rfid
main PPT.pptx of girls hostel security using rfidNikhilNagaraju
 
Heart Disease Prediction using machine learning.pptx
Heart Disease Prediction using machine learning.pptxHeart Disease Prediction using machine learning.pptx
Heart Disease Prediction using machine learning.pptxPoojaBan
 
VIP Call Girls Service Kondapur Hyderabad Call +91-8250192130
VIP Call Girls Service Kondapur Hyderabad Call +91-8250192130VIP Call Girls Service Kondapur Hyderabad Call +91-8250192130
VIP Call Girls Service Kondapur Hyderabad Call +91-8250192130Suhani Kapoor
 
Microscopic Analysis of Ceramic Materials.pptx
Microscopic Analysis of Ceramic Materials.pptxMicroscopic Analysis of Ceramic Materials.pptx
Microscopic Analysis of Ceramic Materials.pptxpurnimasatapathy1234
 

Recently uploaded (20)

Application of Residue Theorem to evaluate real integrations.pptx
Application of Residue Theorem to evaluate real integrations.pptxApplication of Residue Theorem to evaluate real integrations.pptx
Application of Residue Theorem to evaluate real integrations.pptx
 
Software and Systems Engineering Standards: Verification and Validation of Sy...
Software and Systems Engineering Standards: Verification and Validation of Sy...Software and Systems Engineering Standards: Verification and Validation of Sy...
Software and Systems Engineering Standards: Verification and Validation of Sy...
 
SPICE PARK APR2024 ( 6,793 SPICE Models )
SPICE PARK APR2024 ( 6,793 SPICE Models )SPICE PARK APR2024 ( 6,793 SPICE Models )
SPICE PARK APR2024 ( 6,793 SPICE Models )
 
★ CALL US 9953330565 ( HOT Young Call Girls In Badarpur delhi NCR
★ CALL US 9953330565 ( HOT Young Call Girls In Badarpur delhi NCR★ CALL US 9953330565 ( HOT Young Call Girls In Badarpur delhi NCR
★ CALL US 9953330565 ( HOT Young Call Girls In Badarpur delhi NCR
 
HARMONY IN THE HUMAN BEING - Unit-II UHV-2
HARMONY IN THE HUMAN BEING - Unit-II UHV-2HARMONY IN THE HUMAN BEING - Unit-II UHV-2
HARMONY IN THE HUMAN BEING - Unit-II UHV-2
 
Model Call Girl in Narela Delhi reach out to us at 🔝8264348440🔝
Model Call Girl in Narela Delhi reach out to us at 🔝8264348440🔝Model Call Girl in Narela Delhi reach out to us at 🔝8264348440🔝
Model Call Girl in Narela Delhi reach out to us at 🔝8264348440🔝
 
Biology for Computer Engineers Course Handout.pptx
Biology for Computer Engineers Course Handout.pptxBiology for Computer Engineers Course Handout.pptx
Biology for Computer Engineers Course Handout.pptx
 
Gurgaon ✡️9711147426✨Call In girls Gurgaon Sector 51 escort service
Gurgaon ✡️9711147426✨Call In girls Gurgaon Sector 51 escort serviceGurgaon ✡️9711147426✨Call In girls Gurgaon Sector 51 escort service
Gurgaon ✡️9711147426✨Call In girls Gurgaon Sector 51 escort service
 
GDSC ASEB Gen AI study jams presentation
GDSC ASEB Gen AI study jams presentationGDSC ASEB Gen AI study jams presentation
GDSC ASEB Gen AI study jams presentation
 
9953056974 Call Girls In South Ex, Escorts (Delhi) NCR.pdf
9953056974 Call Girls In South Ex, Escorts (Delhi) NCR.pdf9953056974 Call Girls In South Ex, Escorts (Delhi) NCR.pdf
9953056974 Call Girls In South Ex, Escorts (Delhi) NCR.pdf
 
Call Girls Delhi {Jodhpur} 9711199012 high profile service
Call Girls Delhi {Jodhpur} 9711199012 high profile serviceCall Girls Delhi {Jodhpur} 9711199012 high profile service
Call Girls Delhi {Jodhpur} 9711199012 high profile service
 
🔝9953056974🔝!!-YOUNG call girls in Rajendra Nagar Escort rvice Shot 2000 nigh...
🔝9953056974🔝!!-YOUNG call girls in Rajendra Nagar Escort rvice Shot 2000 nigh...🔝9953056974🔝!!-YOUNG call girls in Rajendra Nagar Escort rvice Shot 2000 nigh...
🔝9953056974🔝!!-YOUNG call girls in Rajendra Nagar Escort rvice Shot 2000 nigh...
 
HARMONY IN THE NATURE AND EXISTENCE - Unit-IV
HARMONY IN THE NATURE AND EXISTENCE - Unit-IVHARMONY IN THE NATURE AND EXISTENCE - Unit-IV
HARMONY IN THE NATURE AND EXISTENCE - Unit-IV
 
VICTOR MAESTRE RAMIREZ - Planetary Defender on NASA's Double Asteroid Redirec...
VICTOR MAESTRE RAMIREZ - Planetary Defender on NASA's Double Asteroid Redirec...VICTOR MAESTRE RAMIREZ - Planetary Defender on NASA's Double Asteroid Redirec...
VICTOR MAESTRE RAMIREZ - Planetary Defender on NASA's Double Asteroid Redirec...
 
microprocessor 8085 and its interfacing
microprocessor 8085  and its interfacingmicroprocessor 8085  and its interfacing
microprocessor 8085 and its interfacing
 
Gfe Mayur Vihar Call Girls Service WhatsApp -> 9999965857 Available 24x7 ^ De...
Gfe Mayur Vihar Call Girls Service WhatsApp -> 9999965857 Available 24x7 ^ De...Gfe Mayur Vihar Call Girls Service WhatsApp -> 9999965857 Available 24x7 ^ De...
Gfe Mayur Vihar Call Girls Service WhatsApp -> 9999965857 Available 24x7 ^ De...
 
main PPT.pptx of girls hostel security using rfid
main PPT.pptx of girls hostel security using rfidmain PPT.pptx of girls hostel security using rfid
main PPT.pptx of girls hostel security using rfid
 
Heart Disease Prediction using machine learning.pptx
Heart Disease Prediction using machine learning.pptxHeart Disease Prediction using machine learning.pptx
Heart Disease Prediction using machine learning.pptx
 
VIP Call Girls Service Kondapur Hyderabad Call +91-8250192130
VIP Call Girls Service Kondapur Hyderabad Call +91-8250192130VIP Call Girls Service Kondapur Hyderabad Call +91-8250192130
VIP Call Girls Service Kondapur Hyderabad Call +91-8250192130
 
Microscopic Analysis of Ceramic Materials.pptx
Microscopic Analysis of Ceramic Materials.pptxMicroscopic Analysis of Ceramic Materials.pptx
Microscopic Analysis of Ceramic Materials.pptx
 

Vcs slides on or 2014

  • 1. 2-1 Operations Research Books: Operations Research : An introduction by Hamdy A Taha Ninth ed Pearson
  • 2. 2-2 Today, optimization methods are used everywhere in business, industry, government, engineering, and computer science. You will find optimization taught in various departments of engineering, computer science, business, mathematics, and economics;
  • 3. The desire for optimality (perfection) is inherent for humans Optimization techniques used today have trace their origins during World War II to improve the effectiveness of the war efforts For example: What is the optimum allocation of gasoline supplies among competing campaigns? What is the best search and bombing pattern for anti-submarine patrols? How to manage in the best way the limited food. 2-3
  • 4. 2-4 Here are a few more examples of optimization problems: ·How should the transistors and other devices be laid out in a new computer chip so that the layout takes up the least area? ·What is the smallest number of warehouses, and where should they be located so that the maximum travel time from any retail sales outlet to the closest warehouse is less than 6 hours? ·How should telephone calls be routed between two cities to permit the maximum number of simultaneous calls?
  • 5. 2-5
  • 6. 2-6 A schematic view of modeling/optimization process Real-world problem Mathematical model Solution to model Solution to real-world problem assumptions, abstraction, data simplifications optimization algorithm interpretation makes sense? change the model, assumptions?
  • 7. 2-7 Types of Optimization Models Stochastic (probabilistic information on data) Deterministic (data are certain) Discrete, Integer (S = Zn) Continuous (S = Rn) Linear (f and g are linear) Nonlinear (f and g are nonlinear)
  • 8. 2-8 What is Optimization process?  Optimization is an iterative process by which a desired solution (max/min) of the problem can be found while satisfying all its constraint or bounded conditions.  Optimization problem could be linear or non-linear.  The search procedure is termed as algorithm.
  • 9. 2-9 Mathematical models in Optimization  The general form of an optimization model: min or max f(x1,…,xn) (objective function) subject to gi(x1,…,xn) ≥ 0 (functional constraints) x1,…,xn Î S (set constraints)  x1,…,xn are called decision variables  In words, the goal is to find  --- x1,…,xn that satisfy the constraints; --- achieve min (max) objective function value.
  • 10. 2-10 Optimization Problems Page 10 Unconstrained optimization min{f(x) : x Î Rn} Constrained optimization min{f(x) : ci(x) ≤ 0, i Î I, cj(x) = 0, j Î E} Quadratic programming Tx ≤ bi, i Î I, ai min{1/2xTQx + cTx : ai Tx = bj, j Î E} Zero-One programming min{cTx : Ax = b, x Î {0,1}n, c Î Rn, b Î Rm} Mixed Integer Programming min{cTx : Ax ≤ b, x ≥ 0, xi Î Zn, i Î I, xr Î Rn, r Î R}
  • 11. 2-11 Linear programming is the most widely used of the major techniques for constrained optimization. “Programming” here does not imply computer programming –it is an old word for “planning”. In linear programming, all of the underlying models of the real-world processes are linear, hence you can think of “linear programming” as “planning using linear models”
  • 12. 2-12 = £ å= a x b i m ( 1, 2, ..., ) 1 x j n 12 Introduction  Problems of this kind are called “linear programming problems” or “LP problems” for short; linear programming is the branch of applied mathematics concerned with these problems.  A linear programming problem is the problem of maximizing (or minimizing) a linear function subject to a finite number of linear constraints.  Standard form: å= n j j j c x 1 0 ( 1, 2, ..., ) j i n j ij j ³ = maximize subject to
  • 13. 2-13 History of Linear Programming 13  It started in 1947 when G.B.Dantzig design the “simplex method” for solving linear programming formulations of U.S. Air Force planning problems.  It soon became clear that a surprisingly wide range of apparently unrelated problems in production management could be stated in linear programming terms and solved by the simplex method.  Later, it was used to solve problems of management. It’s algorithm can also used to network flow problems.
  • 14.  On Oct.14th,1975, the Royal Sweden Academy of Science awarded the Nobel Prize in economic science to L.V.Kantorovich and T.C.Koopmans ”for their contributions to the theory of optimum allocation of resources” 2-14 History of Linear Programming  The breakthrough in looking for a theoretically satisfactory algorithm to solve LP problems came in 1979 when L.G.Khachian published a description of such an algorithm. 14
  • 15. 2-15 15 Applications  Efficient allocation of scarce resources  diet problem  Scheduling production and inventory  multistage scheduling problems  The cutting-stock problem  find a way to cut paper or textiles rolls by complicated summary of orders  Approximating data by linear functions  find approximate solutions to possibly unsolvable systems of linear equations
  • 16. 2-16  Model Formulation  A Maximization Model Example  Graphical Solutions of Linear Programming Models  A Minimization Model Example  Irregular Types of Linear Programming Models  Characteristics of Linear Programming Problems
  • 17. Linear Programming: An Overview  Objectives of business decisions frequently involve maximizing profit or minimizing costs.  Linear programming uses linear algebraic relationships to represent a firm’s decisions, given a business objective, and resource constraints.  Steps in application: 1. Identify problem as solvable by linear programming. 2. Formulate a mathematical model of the unstructured 2-17 problem. 3. Solve the model. 4. Implementation
  • 18. 2-18 Properties of Linear Programming Models • Proportionality - The rate of change (slope) of the objective function and constraint equations is constant. • Additivity - Terms in the objective function and constraint equations must be additive. • Divisibility -Decision variables can take on any fractional value and are therefore continuous as opposed to integer in nature. • Certainty - Values of all the model parameters are assumed to be known with certainty (non-probabilistic).
  • 19. 2-19 Model Components  Decision variables - mathematical symbols representing levels of activity of a firm.  Objective function - a linear mathematical relationship describing an objective of the firm, in terms of decision variables - this function is to be maximized or minimized.  Constraints – requirements or restrictions placed on the firm by the operating environment, stated in linear relationships of the decision variables.  Parameters - numerical coefficients and constants used in the objective function and constraints.
  • 20. 2-20 Summary of Model Formulation Steps Step 1 : Clearly define the decision variables Step 2 : Construct the objective function Step 3 : Formulate the constraints
  • 21. 2-21 21 Introduction  A Diet Problem eg: Polly wonders how much money she must spend on food in order to get all the energy (1,000 kcal ), protein (55 g), and calcium (800 mg) that she needs every day. She choose six foods that seem to be cheap sources of the nutrients: Food Serving size Energy (kcal) Protein (g) Calcium (mg) Price per serving (c) Oatmeal 28 g 110 4 2 3 Chicken 100 g 205 32 12 24 Eggs 2 large 160 13 54 13 Whole Milk 237 cc 160 8 285 9 Cherry pie 170 g 420 4 22 20 Pork with beans 260 g 260 14 80 19
  • 22. 2-22 22 Introduction  Servings-per-day limits on all six foods: Oatmeal at most 4 servings per day Chicken at most 3 servings per day Eggs at most 2 servings per day Milk at most 8 servings per day Cherry pie at most 2 servings per day Pork with beans at most 2 servings per day  Now there are so many combinations seem promising that one could go on and on, looking for the best one. Trial and error is not particularly helpful here.
  • 23. 2-23 1 2 3 4 5 6 minimize 3x + 24x + 13x + 9x + 20x + 19x subject to x x x x x x + + + + + ³ 110 205 160 160 420 260 1,000 1 2 3 4 5 6 x x x x x x + + + + + ³ 4 32 13 8 4 14 55 1 2 3 4 5 6 x x x x x x + + + + + ³ 23 Introduction  A new way to express this—using inequalities: x £ £ 0 4 1 x £ £ 0 3 2 x £ £ 0 2 3 £ £ 0 8 4 x £ £ 0 2 5 x £ £ 0 2 6 x 2 12 54 285 22 80 800 1 2 3 4 5 6
  • 24. 2-24 = £ å= a x b i m ( 1, 2, ..., ) 1 x j n 24 Introduction  Problems of this kind are called “linear programming problems” or “LP problems” for short; linear programming is the branch of applied mathematics concerned with these problems.  A linear programming problem is the problem of maximizing (or minimizing) a linear function subject to a finite number of linear constraints.  Standard form: å= n j j j c x 1 0 ( 1, 2, ..., ) j i n j ij j ³ = maximize subject to
  • 25. 2-25 LP Model Formulation A Maximization Example (1 of 4)  Product mix problem - Beaver Creek Pottery Company  How many bowls and mugs should be produced to maximize profits given labor and materials constraints?  Product resource requirements and unit profit: Resource Requirements Product Labor (Hr./Unit) Clay (Lb./Unit) Profit ($/Unit) Bowl 1 4 40 Mug 2 3 50
  • 26. 2-26 LP Model Formulation A Maximization Example (2 of 4)
  • 27. 2-27 LP Model Formulation A Maximization Example (3 of 4) Resource 40 hrs of labor per day Availability: 120 lbs of clay Decision x1 = number of bowls to produce per day Variables: x2 = number of mugs to produce per day Objective Maximize Z = $40x1 + $50x2 Function: Where Z = profit per day Resource 1x1 + 2x2 £ 40 hours of labor Constraints: 4x1 + 3x2 £ 120 pounds of clay Non-Negativity x1 ³ 0; x2 ³ 0 Constraints:
  • 28. 2-28 LP Model Formulation A Maximization Example (4 of 4) Complete Linear Programming Model: Maximize Z = $40x1 + $50x2 subject to: 1x1 + 2x2 £ 40 4x1 + 3x2 £ 120 x1, x2 ³ 0
  • 29. Feasible Solutions A feasible solution does not violate any of the constraints: Example: x1 = 5 bowls 2-29 x2 = 10 mugs Z = $40x1 + $50x2 = $700 Labor constraint check: 1(5) + 2(10) = 25 < 40 hours Clay constraint check: 4(5) + 3(10) = 50 < 120 pounds
  • 30. 2-30 Infeasible Solutions An infeasible solution violates at least one of the constraints: Example: x1 = 10 bowls x2 = 20 mugs Z = $40x1 + $50x2 = $1400 Labor constraint check: 1(10) + 2(20) = 50 > 40 hours
  • 31. 2-31 Graphical Solution of LP Models  Graphical solution is limited to linear programming models containing only two decision variables (can be used with three variables but only with great difficulty).  Graphical methods provide visualization of how a solution for a linear programming problem is obtained.
  • 32. Coordinate Axes Graphical Solution of Maximization Model (1 of 12) 2-32 Maximize Z = $40x1 + $50x2 subject to: 1x1 + 2x2 £ 40 4x1 + 3x2 £ 120 x1, x2 ³ 0 Figure Coordinates for Graphical Analysis X1 is bowls X2 is mugs
  • 33. Labor Constraint Graphical Solution of Maximization Model (2 of 12) 2-33 Figure Graph of Labor Constraint Maximize Z = $40x1 + $50x2 subject to: 1x1 + 2x2 £ 40 4x1 + 3x2 £ 120 x1, x2 ³ 0
  • 34. Labor Constraint Area Graphical Solution of Maximization Model (3 of 12) 2-34 Figure Labor Constraint Area Maximize Z = $40x1 + $50x2 subject to: 1x1 + 2x2 £ 40 4x1 + 3x2 £ 120 x1, x2 ³ 0
  • 35. Clay Constraint Area Graphical Solution of Maximization Model (4 of 12) 2-35 Figure Clay Constraint Area Maximize Z = $40x1 + $50x2 subject to: 1x1 + 2x2 £ 40 4x1 + 3x2 £ 120 x1, x2 ³ 0
  • 36. Both Constraints Graphical Solution of Maximization Model (5 of 12) 2-36 Maximize Z = $40x1 + $50x2 subject to: 1x1 + 2x2 £ 40 4x1 + 3x2 £ 120 x1, x2 ³ 0 Figure Graph of Both Model Constraints
  • 37. Feasible Solution Area Graphical Solution of Maximization Model (6 of 12) 2-37 Figure Feasible Solution Area Maximize Z = $40x1 + $50x2 subject to: 1x1 + 2x2 £ 40 4x1 + 3x2 £ 120 x1, x2 ³ 0
  • 38. Objective Function Solution = $800 Graphical Solution of Maximization Model (7 of 12) 2-38 Maximize Z = $40x1 + $50x2 subject to: 1x1 + 2x2 £ 40 4x1 + 3x2 £ 120 x1, x2 ³ 0 Figure Objection Function Line for Z = $800
  • 39. 2-39 Alternative Objective Function Solution Lines Graphical Solution of Maximization Model (8 of 12) Maximize Z = $40x1 + $50x2 subject to: 1x1 + 2x2 £ 40 4x1 + 3x2 £ 120 x1, x2 ³ 0 Figure Alternative Objective Function Lines
  • 40. Optimal Solution Graphical Solution of Maximization Model (9 of 12) 2-40 Maximize Z = $40x1 + $50x2 subject to: 1x1 + 2x2 £ 40 4x1 + 3x2 £ 120 x1, x2 ³ 0 Figure Identification of Optimal Solution Point
  • 41. 2-41 Optimal Solution Coordinates Graphical Solution of Maximization Model (10 of 12) Figure Optimal Solution Coordinates Maximize Z = $40x1 + $50x2 subject to: 1x1 + 2x2 £ 40 4x1 + 3x2 £ 120 x1, x2 ³ 0
  • 42. 2-42 Extreme (Corner) Point Solutions Graphical Solution of Maximization Model (11 of 12) Figure Solutions at All Corner Points Maximize Z = $40x1 + $50x2 subject to: 1x1 + 2x2 £ 40 4x1 + 3x2 £ 120 x1, x2 ³ 0
  • 43. 2-43 Optimal Solution for New Objective Function Graphical Solution of Maximization Model (12 of 12) Maximize Z = $70x1 + $20x2 subject to: 1x1 + 2x2 £ 40 4x1 + 3x2 £ 120 x1, x2 ³ 0 Figure Optimal Solution with Z = 70x1 + 20x2
  • 44.  Standard form requires that all constraints be in the form of equations (equalities).  A slack variable is added to a £ constraint (weak inequality) to convert it to an equation (=).  A slack variable typically represents an unused resource.  A slack variable contributes nothing to the objective function value. 2-44 Slack Variables
  • 45. Linear Programming Model: Standard Form 2-45 Max Z = 40x1 + 50x2 + 0s1 + 0s2 subject to:1x1 + 2x2 + s1 = 40 4x1 + 3x2 + s2 = 120 x1, x2, s1, s2 ³ 0 Where: x1 = number of bowls x2 = number of mugs s1, s2 are slack variables Figure Solution Points A, B, and C with Slack
  • 46. 2-46 LP Model Formulation – Minimization (1 of 8)  Two brands of fertilizer available - Super-gro, Crop-quick.  Field requires at least 16 pounds of nitrogen and 24 pounds of phosphate.  Super-gro costs $6 per bag, Crop-quick $3 per bag.  Problem: How much of each brand to purchase to minimize total cost of fertilizer given following data ? Chemical Contribution Brand Nitrogen (lb/bag) Phosphate (lb/bag) Super-gro 2 4 Crop-quick 4 3
  • 47. 2-47 LP Model Formulation – Minimization (2 of 8) Figure Fertilizing farmer’s field
  • 48. 2-48 LP Model Formulation – Minimization (3 of 8) Decision Variables: x1 = bags of Super-gro x2 = bags of Crop-quick The Objective Function: Minimize Z = $6x1 + 3x2 Where: $6x1 = cost of bags of Super-Gro $3x2 = cost of bags of Crop-Quick Model Constraints: 2x1 + 4x2 ³ 16 lb (nitrogen constraint) 4x1 + 3x2 ³ 24 lb (phosphate constraint) x1, x2 ³ 0 (non-negativity constraint)
  • 49. 2-49 Constraint Graph – Minimization (4 of 8) Minimize Z = $6x1 + $3x2 subject to: 2x1 + 4x2 ³ 16 4x1 + 3x2 ³ 24 x1, x2 ³ 0 Figure Graph of Both Model Constraints
  • 50. Feasible Region– Minimization (5 of 8) Figure Feasible Solution Area Minimize Z = $6x1 + $3x2 subject to: 2x1 + 4x2 ³ 16 4x1 + 3x2 ³ 24 x1, x2 ³ 0 Copyright © 2010 Pearson Education, Inc. Publishing as Prentice Hall 2-50
  • 51. 2-51 Optimal Solution Point – Minimization (6 of 8) Figure Optimum Solution Point Minimize Z = $6x1 + $3x2 subject to: 2x1 + 4x2 ³ 16 4x1 + 3x2 ³ 24 x1, x2 ³ 0
  • 52. Surplus Variables – Minimization (7 of 8)  A surplus variable is subtracted from a ³ constraint to convert it to an equation (=).  A surplus variable represents an excess above a constraint requirement level.  A surplus variable contributes nothing to the calculated value of the objective function.  Subtracting surplus variables in the farmer problem constraints: 2-52 2x1 + 4x2 - s1 = 16 (nitrogen) 4x1 + 3x2 - s2 = 24 (phosphate)
  • 53. 2-53 Graphical Solutions – Minimization (8 of 8) Minimize Z = $6x1 + $3x2 + 0s1 + 0s2 subject to: 2x1 + 4x2 – s1 = 16 4x1 + 3x2 – s2 = 24 x1, x2, s1, s2 ³ 0 Figure Graph of Fertilizer Example
  • 54. Irregular Types of Linear Programming Problems 2-54 For some linear programming models, the general rules do not apply.  Special types of problems include those with:  Multiple optimal solutions  Infeasible solutions  Unbounded solutions
  • 55. 2-55 Multiple Optimal Solutions Beaver Creek Pottery The objective function is parallel to a constraint line. Maximize Z=$40x1 + 30x2 subject to: 1x1 + 2x2 £ 40 4x1 + 3x2 £ 120 x1, x2 ³ 0 Where: x1 = number of bowls x2 = number of mugs Figure Example with Multiple Optimal Solutions
  • 56. 2-56 An Infeasible Problem Every possible solution violates at least one constraint: Maximize Z = 5x1 + 3x2 subject to: 4x1 + 2x2 £ 8 Figure Graph of an Infeasible Problem x1 ³ 4 x2 ³ 6 x1, x2 ³ 0
  • 57. 2-57 An Unbounded Problem Value of the objective function increases indefinitely: Maximize Z = 4x1 + 2x2 subject to: x1 ³ 4 Figure Graph of an Unbounded Problem x2 £ 2 x1, x2 ³ 0
  • 58. 2-58 Characteristics of Linear Programming Problems  A decision amongst alternative courses of action is required.  The decision is represented in the model by decision variables.  The problem encompasses a goal, expressed as an objective function, that the decision maker wants to achieve.  Restrictions (represented by constraints) exist that limit the extent of achievement of the objective.  The objective and constraints must be definable by linear mathematical functional relationships.
  • 59. 2-59 Properties of Linear Programming Models  Proportionality - The rate of change (slope) of the objective function and constraint equations is constant.  Additivity - Terms in the objective function and constraint equations must be additive.  Divisibility -Decision variables can take on any fractional value and are therefore continuous as opposed to integer in nature.  Certainty - Values of all the model parameters are assumed to be known with certainty (non-probabilistic).
  • 60. 2-60 Problem Statement Example Problem No. 1 (1 of 3) ■ Hot dog mixture in 1000-pound batches. ■ Two ingredients, chicken ($3/lb) and beef ($5/lb). ■ Recipe requirements: at least 500 pounds of “chicken” at least 200 pounds of “beef” ■ Ratio of chicken to beef must be at least 2 to 1. ■ Determine optimal mixture of ingredients that will minimize costs.
  • 61. 2-61 Solution Example Problem No. 1 (2 of 3) Step 1: Identify decision variables. x1 = lb of chicken in mixture x2 = lb of beef in mixture Step 2: Formulate the objective function. Minimize Z = $3x1 + $5x2 where Z = cost per 1,000-lb batch $3x1 = cost of chicken $5x2 = cost of beef
  • 62. 2-62 Solution Example Problem No. 1 (3 of 3) Step 3: Establish Model Constraints x1 + x2 = 1,000 lb x1 ³ 500 lb of chicken x2 ³ 200 lb of beef x1/x2 ³ 2/1 or x1 - 2x2 ³ 0 x1, x2 ³ 0 The Model: Minimize Z = $3x1 + 5x2 subject to: x1 + x2 = 1,000 lb x1 ³ 500 x2 ³ 200 x1 - 2x2 ³ 0 x,x³ 0
  • 63. 2-63 Solve the following model graphically: Maximize Z = 4x1 + 5x2 subject to: x1 + 2x2 £ 10 6x1 + 6x2 £ 36 x1 £ 4 x1, x2 ³ 0 Step 1: Plot the constraints as equations Figure Constraint Equations
  • 64. 2-64 Maximize Z = 4x1 + 5x2 subject to: x1 + 2x2 £ 10 6x1 + 6x2 £ 36 x1 £ 4 x1, x2 ³ 0 Step 2: Determine the feasible solution space Figure Feasible Solution Space and Extreme Points
  • 65. 2-65 Maximize Z = 4x1 + 5x2 subject to: x1 + 2x2 £ 10 6x1 + 6x2 £ 36 x1 £ 4 x1, x2 ³ 0 Step 3 and 4: Determine the solution points and optimal solution Figure Optimal Solution Point
  • 66. 2-66 Simplex Method  SSiimmpplleexx MMeetthhoodd  CChhaarraacctteerriissttiiccss ooff SSiimmpplleexx MMeetthhoodd  WWhhyy wwee sshhoouulldd ssttuuddyy tthhee SSiimmpplleexx MMeetthhoodd??  SSuummmmaarryy ooff tthhee SSiimmpplleexx MMeetthhoodd  EExxaammpplleess ssoollvveedd bbyy ccoonndduuccttiinngg ttaabbuullaarr mmeetthhoodd 66
  • 67. SIMPLEX METHOD  Linear programming mmooddeellss ccoouulldd bbee ssoollvveedd aallggeebbrraaiiccaallllyy..  TThhee mmoosstt wwiiddeellyy uusseedd aallggeebbrraaiicc pprroocceedduurree ffoorr ssoollvviinngg lliinneeaarr pprrooggrraammmmiinngg pprroobblleemm iiss ccaalllleedd tthhee SSiimmpplleexx MMeetthhoodd..  TThhee ssiimmpplleexx mmeetthhoodd iiss aa ggeenneerraall--ppuurrppoossee lliinneeaarr-- pprrooggrraammmmiinngg aallggoorriitthhmm wwiiddeellyy uusseedd ttoo ssoollvvee llaarrggee ssccaallee pprroobblleemmss.. AAlltthhoouugghh iitt llaacckkss tthhee iinnttuuiittiivvee aappppeeaall ooff tthhee ggrraapphhiiccaall aapppprrooaacchh,, iittss aabbiilliittyy ttoo hhaannddllee pprroobblleemmss wwiitthh mmoorree tthhaann ttwwoo ddeecciissiioonn vvaarriiaabblleess mmaakkeess iitt eexxttrreemmeellyy vvaalluuaabbllee ffoorr ssoollvviinngg pprroobblleemmss oofftteenn eennccoouunntteerreedd iinn pprroodduuccttiioonn//ooppeerraattiioonnss mmaannaaggeemmeenntt..  TThhuuss ssiimmpplleexx mmeetthhoodd ooffffeerrss aann eeffffiicciieenntt mmeeaannss ooff ssoollvviinngg mmoorree ccoommpplleexx lliinneeaarr pprrooggrraammmmiinngg pprroobblleemmss.. 2-67 67
  • 68. 2-68 Characteristics of Simplex Method  In the simplex method, the computational rroouuttiinnee iiss aann iitteerraattiivvee pprroocceessss.. TToo iitteerraattee mmeeaannss ttoo rreeppeeaatt;; hheennccee,, iinn wwoorrkkiinngg ttoowwaarrdd tthhee ooppttiimmuumm ssoolluuttiioonn,, tthhee ccoommppuuttaattiioonnaall rroouuttiinnee iiss rreeppeeaatteedd oovveerr aanndd oovveerr,, ffoolllloowwiinngg aa ssttaannddaarrdd ppaatttteerrnn..  SSuucccceessssiivvee ssoolluuttiioonnss aarree ddeevveellooppeedd iinn aa ssyysstteemmaattiicc ppaatttteerrnn uunnttiill tthhee bbeesstt ssoolluuttiioonn iiss rreeaacchheedd..  EEaacchh nneeww ssoolluuttiioonn wwiillll yyiieelldd aa vvaalluuee ooff tthhee oobbjjeeccttiivvee ffuunnccttiioonn aass llaarrggee aass oorr llaarrggeerr tthhaann tthhee pprreevviioouuss ssoolluuttiioonn.. TThhiiss iimmppoorrttaanntt ffeeaattuurree aassssuurreess uuss tthhaatt wwee aarree aallwwaayyss mmoovviinngg cclloosseerr ttoo tthhee ooppttiimmuumm aannsswweerr.. FFiinnaallllyy,, tthhee mmeetthhoodd iinnddiiccaatteess wwhheenn tthhee ooppttiimmuumm ssoolluuttiioonn hhaass bbeeeenn rreeaacchheedd.. 68
  • 69. 2-69 Characteristics of Simplex Method  lif Most real-lifee lliinneeaarr pprrooggrraammmmiinngg pprroobblleemmss hhaavvee mmoorree tthhaann ttwwoo vvaarriiaabblleess,, ssoo aa pprroocceedduurree ccaalllleedd tthhee ssiimmpplleexx mmeetthhoodd iiss uusseedd ttoo ssoollvvee ssuucchh pprroobblleemmss.. TThhiiss pprroocceedduurree ssoollvveess tthhee pprroobblleemm iinn aann iitteerraattiivvee mmaannnneerr,, tthhaatt iiss,, rreeppeeaattiinngg tthhee ssaammee sseett ooff pprroocceedduurreess ttiimmee aafftteerr ttiimmee uunnttiill aann ooppttiimmaall ssoolluuttiioonn iiss rreeaacchheedd.. EEaacchh iitteerraattiioonn bbrriinnggss aa hhiigghheerr vvaalluuee ffoorr tthhee oobbjjeeccttiivvee ffuunnccttiioonn ssoo tthhaatt wwee aarree aallwwaayyss mmoovviinngg cclloosseerr ttoo tthhee ooppttiimmaall ssoolluuttiioonn.. 69
  • 70. Characteristics of Simplex Method The simplex method requires simple mathematical operations (addition, subtraction, multiplication, and division), but the computations are lengthy and tedious, and the slightest error can lead to a good deal of frustration. For these reasons, most users of the technique rely on computers to handle the computations while they concentrate on the solutions. Still, some familiarity with manual computations is helpful in understanding the simplex process. The student will discover that it is better not to use his/her calculator in working through these problems because rounding can easily distort the results. Instead, it is better to work with numbers in fractional form. 2-70 70
  • 71. • It is important to understand the ideas used to produce solution. The simplex approach yields not only the optimal solution to the xi variables, and the maximum profit (or minimum cost) but valuable economic information as well. • To be able to use computers successfully and to interpret LP computer print outs, we need to know what the simplex method is doing and why. • We begin by solving a maximization problem using the simplex method. We then tackle a minimization problem. 2-71 Why we should study the Simplex Method? 71
  • 72. Step 1. Formulate a LP model of the problem. Step 2. Add slack variables to each constraint to obtain standard form. Step 3. Set up the initial simplex tableau. Step 4. Choose the nonbasic variable with the largest entry in the net evaluation row (Cj – Zj) to bring into the basis. This identifies the pivot (key) column; the column associated with the incoming variable. Step 5. Choose as the pivot row that row with the smallest ratio of “bi/ aij”, for aij >0 2-72 SUMMARY OF THE SIMPLEX METHOD where j is the pivot column. This identifies the pivot row, the row of the variable leaving the basis when variable j enters. Step 6. a). Divide each element of the pivot row by the pivot element. b). According to the entering variable, find the new values for remaining 72 variables. Step 7. Test for optimality. If Cj – Zj £ 0 for all columns, we have the optimal solution. If not, return to step 4.
  • 73. 2-73 73 Example 1 A Furniture Ltd., wants to determine the most profitable combination of products to manufacture given that its resources are limited. The Furniture Ltd., makes two products, tables and chairs, which must be processed through assembly and finishing departments. Assembly has 60 hours available; Finishing can handle up to 48 hours of work. Manufacturing one table requires 4 hours in assembly and 2 hours in finishing. Each chair requires 2 hours in assembly and 4 hours in finishing. Profit is $8 per table and $6 per chair.
  • 74. 2-74 Tabular solution for Example 1/1 74
  • 75. 2-75 Tabular solution for Example 1/2 75
  • 76. 2-76 Tabular solution for Example 1/3 76
  • 77. 2-77 Tabular solution for Example 1/4 77
  • 78. 2-78 Tabular solution for Example 1/5 78
  • 79. 2-79 Tabular solution for Example 1/6 79
  • 80. 2-80 Tabular solution for Example 1/7 80
  • 81. 2-81 Tabular solution for Example 1/8 81
  • 82. Example 2 PAR Inc. produces golf equipment and decided to move into the market for standard Cutting and dyeing the material, Sewing, Finishing (inserting umbrella holder, club separators etc.), Inspection and packaging. Each standard golf-bag will require 7/10 hr. in the cutting and dyeing department, 1/2 hr. in the sewing department, 1 hr. in the finishing department and 1/10 hr. in the inspection & packaging department. Deluxe model will require 1 hr. in the cutting and dyeing department, 5/6 hr. for The profit contribution for every standard bag is 10 MU and for every deluxe bag is 9 2-82 and deluxe golf bags. Each golf bag requires the following operations: sewing, 2/3 hr. for finishing and 1/4 hr. for inspection and packaging 82 MU. In addition the total hours available during the next 3 months are as follows: Cutting & dyeing dept 630 hrs Sewing dept 600 hrs Finishing 708 hrs Inspection & packaging 135 hrs The company’s problem is to determine how many standard and deluxe bags should be produced in the next 3 months?
  • 84. 2-84 Tabular solution for Example 2/1 84
  • 85. 2-85 Tabular solution for Example 2/2 85
  • 86. 2-86 Tabular solution for Example 2/3 86
  • 87. 2-87 Tabular solution for Example 2/4 87
  • 88. 2-88 Tabular solution for Example 2/5 88
  • 89. 2-89 89 Example 3 High Tech industries import components for production of two different models of personal computers, called deskpro and portable. High Tech’s management is currently interested in developing a weekly production schedule for both products. The deskpro generates a profit contribution of $50/unit, and portable generates a profit contribution of $40/unit. For next week’s production, a max of 150 hours of assembly time is available. Each unit of deskpro requires 3 hours of assembly time. And each unit of portable requires 5 hours of assembly time. High Tech currently has only 20 portable display components in inventory; thus no more than 20 units of portable may be assembled. Only 300 sq. feet of warehouse space can be made available for new production. Assembly of each Deskpro requires 8 sq. ft. of warehouse space, and each Portable requires 5 sq. ft. of warehouse space.
  • 90. 2-90 Tabular solution for Example 3/1 90
  • 91. 2-91 Tabular solution for Example 3/2 91
  • 92. 2-92 Tabular solution for Example 3/3 92
  • 93. 2-93 Tabular solution for Example 3/3 93
  • 94. 2-94 Tableau Form : The Special Case  Obtaining tableau form is somewhat more complex if the LP contains ³ constraints, = constraints, and/or “-ve” right-hand-side values. Here we will explain how to develop tableau form for each of these situations. 94
  • 95.  Suppose that in the high-tech industries problem, management wanted to ensure that the combined total production for both models would be at least 25 units.  Thus,  Objective Function Max Z = 50X1 + 40X2  1X1 £ 20 Portable display  8X1 + 5 X2 £ 300 Warehouse space  1X1 + 1X2 ³ 25 Min. total production  X1 , X2 ³ 0 2-95 Tabular solution for Example 4/1 Subjective to : 3 X1 + 5 X2 £ 150 Assembly time 95
  • 96.  First, we use three slack variables and one surplus variable to write the problem in std. Form.  Max Z = 50X1 + 40X2 + 0S1 + 0S2 + 0S3 + 0S4  Subject to 3X1 + 5X2 + 1S1 = 150  1X2 + 1S2 = 20  8X1 + 5X2 + 1S3 = 300  1X1 + 1X2 - 1S4 = 25  All variables ³ 0  For the initial tableau X1 = 0 X2 = 0  S1= 150 S2 = 20  S3 = 300 S4 = -25 2-96 Tabular solution for Example 4/2 96
  • 97.  Clearly this is not a basic feasible solution since S4 = -25 violates the nonnegativity requirement. We introduce new variable called ARTIFICIAL VARIABLE.  Artificial variables will be eliminated before the optimal solution is reached. We assign a very large cost to the variable in the objective function. Objective function 50X1 + 40X2 + 0S1 + 0S2 + 0S3 + 0S4 - MA4 2-97 Tabular solution for Example 4/3 97
  • 98. 2-98 Tabular solution for Example 4/4 Initial Tableau Cj 50 40 0 0 0 0 -M Max. (Entering) 98 Product mix Quantit bi X1 X2 S1 S2 S3 S4 A4 bi / aij 0 S1 150 3 5 1 0 0 0 0 150/3=50 0 S2 20 0 1 0 1 0 0 0 -- 0 S3 300 8 5 0 0 1 0 0 300/8=37.5 -M A4 25 1 1 0 0 0 -1 1 25 Min. leaving Zj -25M -M -M 0 0 0 M -M Cj – Zj 50+M 40+M 0 0 0 -M 0 New X1 values = 25, 1, 1, 0, 0, 0, -1, 1
  • 99. 2-99 Tabular solution for Example 4/5 99
  • 100. 2-100 Tabular solution for Example 4/6 2nd Tableau Cj $50 40 0 0 0 0 -M 100 Prodt mix Quant bi X1 X2 S1 S2 S3 S4 A4 bi / aij $0 S1 75 0 2 1 0 0 3 -3 75/3=25 0 S2 20 0 1 0 1 0 0 0 -- 0 S3 100 0 -3 0 0 1 8 -8 100/8=12.5 Min,leaving 50 X1 25 1 1 0 0 0 -1 1 -- Zj $1250 50 50 0 0 0 -50 50 Cj – Zj 0 -10 0 0 0 50 -M-50 Max. (Entering) new S4 values : 100/8 = 25/2, 0, -3/8, 0, 0, 1/8, 1
  • 101. 2-101 Tabular solution for Example 4/7  IMPORTANT!!  Since A4 is an artificial variable that was added simply to obtain an initial basic feasible solution, we can drop its associated column from the simplex tableau.  Indeed whenever artificial variables are used, they can be dropped from the simplex tableau as soon as they have been eliminated from the basic feasible solution. 101
  • 102. 2-102 Tabular solution for Example 4/8 102
  • 103. 2-103 Tabular solution for Example 4/9 One more iteration is required. This time X2 comes into the solution and S1 is eliminated. After performing this iteration, the following simplex tableau shows that the optimal solution has been reached. 103
  • 104. 2-104 104 Tabular solution for Example 4/10
  • 105. EQUALITY CONTRAINTS NEGATIVE RIGHT-HAND SIDE VALUES  Simply add an artificial variable A1 to create a basic feasible solution in the initial simplex tableau. 6X1 + 4X2 - 5X3 = 30 Þ6X1 + 4X2 - 5X3 + 1A1 = 30  One of the properties of the tableau form of a linear program is that the values on the right-hand sides of the constraints have to be nonnegative.  e.g. # of units of the portable model (X2) has to be less than or equal to the # of units of the deskpro model (X1) after setting aside 5 units of the deskpro for internal company use. 2-105 105 X2 £ X1 - 5  - X1 + X2 £ -5  (Min)Multiply by –1 Þ (Max) X1 - X2 ³ 5  We now have an acceptable nonnegative right-hand-side value. Tableau form for this constraint can now be obtained by subtracting a surplus variable and adding an artificial variable.
  • 106.  Livestock Nutrition Co. produces specially blended feed supplements. LNC currently has an order for 200 kgs of its mixture.  This consists of two ingredients X1 ( a protein source ) X2 ( a carbohydrate source )  The first ingredient, X1 costs LNC 3MU a kg. The second ingredient, X2 costs LNC 8MU a kg. The mixture can’t be more than 40% X1 and it must be at least 30% X2.  LNC’s problem is to determine how much of each ingredient to use to minimize cost. 2-106 Tabular solution for Example 5/1 106
  • 107. 2-107 Tabular solution for Example 5/2  The cost function can be written as Cost = 3X1 + 8X2 Min!  LNC must produce 200 kgs of the mixture – no more, no less. X1 + X2 = 200 kgs  The mixture can’t be more than 40% X1, so we may use less than 80 kgs. (40% X 200 = 80). However, we must not exceed 80 kgs. X1 £ 80 kgs  The mixture must be at least 30% X2. Thus we may use more than 60 kgs, not less than 60 kgs. (30% X 200 = 60)  X2 ³ 60 kgs 107
  • 108. 2-108 Tabular solution for Example 5/3 Minimize : Cost = 3MU X1 + 8MU X2 Subject to X1 + X2 = 200 kgs X1 £ 80 kgs X2 ³ 60 kgs X1 , X2 ³ 0  An initial solution: X1 + X2 = 200 kgs Þ X1 + X2 + A1 = 200 108 ß  Artificial variable : A very expensive substance must not be represented in optimal solution.
  • 109. 2-109 Tabular solution for Example 5/4  An artificial Variable is only of value as a computational device; it allows 2 types of restrictions to be treated.  The equality type  ³ type X1 £ 80 kgs constraint on protein Þ X1 + S1 = 80 kgs X2 ³ 60 kgs constraint on carbohydrates Þ X2 - S2 + A2 = 60  X1 , X2 , S1, S2, A1, A2 ³ 0 ß ß ß ß 0MU 0MU M M 109
  • 110. 2-110 Tabular solution for Example 5/5 Minimize : Cost = 3X1 + 8X2 + 0S1 + 0S2 + MA1 + MA2 Subject to : X1 + X2 + A1 = 200 X1 + S1 = 80 X2 - S2 + A2 = 60 110 All variables ³ 0
  • 111. 2-111 Tabular solution for Example 5/6 111
  • 112. 2-112 Tabular solution for Example 5/7 112
  • 113. 2-113 Tabular solution for Example 5/8 113
  • 114. 2-114 Tabular solution for Example 5/9 114
  • 115. 2-115 Tabular solution for Example 5/10 115
  • 116. 2-116 Tabular solution for Example 5/11 116
  • 117. 2-117 Tabular solution for Example 5/12 117
  • 118. Copyright © 2010 Pearson Education, Inc. Publishing as Prentice Hall 2-118
  • 119. Copyright © 2010 Pearson Education, Inc. Publishing as Prentice Hall 2-119
  • 120. B A C D Copyright © 2010 Pearson Education, Inc. Publishing as Prentice Hall 2-120
  • 121. Copyright © 2010 Pearson Education, Inc. Publishing as Prentice Hall 2-121
  • 122. Copyright © 2010 Pearson Education, Inc. Publishing as Prentice Hall 2-122
  • 123. Copyright © 2010 Pearson Education, Inc. Publishing as Prentice Hall 2-123
  • 124. 2-124 Special cases in simplex method  Degeneracy  Alternate optima  Unbounded solution  Nonexisting sol(infeasible) Copyright © 2010 Pearson Education, Inc. Publishing as Prentice Hall
  • 125. 2-125 Degeneracy  At least one basic variable is zero  It occurs when there is a tie for minimum ratio  At least one basic variable will be zero in the next iteration  Degeneracy can cause to cycle indefinitely  This reveals at least one redundant constraint Copyright © 2010 Pearson Education, Inc. Publishing as Prentice Hall
  • 126. 2-126 Example  Max z = 3x + 9 y  S.t. X + 4 y £ 8  x + 2 y £ 4  x, y ³ 0  Solve by simplex and check Copyright © 2010 Pearson Education, Inc. Publishing as Prentice Hall
  • 127. Figure 3.7 LP degeneracy in Example 3.5-1 Copyright © 2010 Pearson Education, Inc. Publishing as Prentice Hall 2-127
  • 128. 2-128 Alternate optima  zero coef of non basic x indicate that x can be made basic,altering the values of the basic variables without changing the value of z.  max z = 2x + 4 y  S.t x + 2 y £ 5  x + y £ 4  x, y ³ 0 Copyright © 2010 Pearson Education, Inc. Publishing as Prentice Hall
  • 129. Figure 3.9 LP alternative optima in Example 3.5-2 Copyright © 2010 Pearson Education, Inc. Publishing as Prentice Hall 2-129
  • 130. 2-130 Unbounded solution  if in the key col all the coeff are non positive  New var can be increased indefinitely without violating any of the constraints  Hence z can be increases indefinitely Copyright © 2010 Pearson Education, Inc. Publishing as Prentice Hall
  • 131. 2-131 example  Max z = 2x + y  S.t x – y £10 2x £ 40 x, y ³ 0 Copyright © 2010 Pearson Education, Inc. Publishing as Prentice Hall
  • 132. Figure 3.10 LP unbounded solution in Example 3.5-3 Copyright © 2010 Pearson Education, Inc. Publishing as Prentice Hall 2-132
  • 133. 2-133 Infeasible sol Case does not occur if all constraints are less than equal to If at least one artificial variable is positive in the optimum iteration the LP has no feasible sol Copyright © 2010 Pearson Education, Inc. Publishing as Prentice Hall
  • 134. 2-134 example  Max z = 3x + 2 y  2x + y £2  3x + 4 y £12  x, y³ 0  Check by solving Copyright © 2010 Pearson Education, Inc. Publishing as Prentice Hall
  • 135. Figure 3.11 Infeasible solution of Example 3.5-4 Copyright © 2010 Pearson Education, Inc. Publishing as Prentice Hall 2-135
  • 136. 2-136 Duality Theory The theory of duality is a very elegant and important concept within the field of operations research.
  • 137. 2-137 The notion of duality within linear programming asserts that every linear program has associated with it a related linear program called its dual. The original problem in relation to its dual is termed the primal.
  • 138. 2-138 Examples There is a small company in Melbourne which has recently become engaged in the production of office furniture. The company manufactures tables, desks and chairs. The production of a table requires 8 kgs of wood and 5 kgs of metal and is sold for $80; a desk uses 6 kgs of wood and 4 kgs of metal and is sold for $60; and a chair requires 4 kgs of both metal and wood and is sold for $50. We would like to determine the revenue maximizing strategy for this company, given that their resources are limited to 100 kgs of wood and 60 kgs of metal.
  • 139. 2-139 Problem P1 Z = 80x1 + 60x2 + 50x3 8 6 4 100 5 4 4 60 max x x x x x x x x x x + + £ + + £ , , ³ 1 2 3 1 2 3 1 2 3 0
  • 140. 2-140 Now consider that there is a much bigger company in Melbourne which has been the lone producer of this type of furniture for many years. They don't appreciate the competition from this new company; so they have decided to tender an offer to buy all of their competitor's resources and therefore put them out of business.
  • 141. The challenge for this large company then is to develop a linear program which will determine the appropriate amount of money that should be offered for a unit of each type of resource, such that the offer will be acceptable to the smaller company while minimizing the expenditures of the larger company. 2-141
  • 142. 2-142 Problem D1 y y y y y y y y + ³ + ³ + ³ , ³ 8 5 80 6 4 60 4 4 50 0 1 2 1 2 1 2 1 2 min y w = 100y1 + 60y2
  • 143. 2-143 A Diet Problem An individual has a choice of two types of food to eat, meat and potatoes, each offering varying degrees of nutritional benefit. He has been warned by his doctor that he must receive at least 400 units of protein, 200 units of carbohydrates and 100 units of fat from his daily diet. Given that a kg of steak costs $10 and provides 80 units of protein, 20 units of carbohydrates and 30 units of fat, and that a kg of potatoes costs $2 and provides 40 units of protein, 50 units of carbohydrates and 20 units of fat, he would like to find the minimum cost diet which satisfies his nutritional requirements
  • 144. 2-144 Problem P2 80 40 400 20 50 200 30 20 100 1 2 1 2 1 2 1 2 0 x x x x x x x x + ³ + ³ + ³ , ³ min x Z =10x1 + 2x2
  • 145. 2-145 Now consider a chemical company which hopes to attract this individual away from his present diet by offering him synthetic nutrients in the form of pills. This company would like determine prices per unit for their synthetic nutrients which will bring them the highest possible revenue while still providing an acceptable dietary alternative to the individual.
  • 146. 2-146 Problem D2 max y w = 400y1 + 200y2 + 100y3 80y1 + 20y2 + 30y3 £10 40y1 + 50y2 + 20y3 £ 2 y1, y2, y3 ³ 0
  • 147. 2-147 Comments Each of the two examples describes some kind of competition between two decision makers. We shall investigate the economic interpretation of the primal/dual relationship later in this chapter.
  • 148. 2-148 FINDING THE DUAL OF A STANDARD LINEAR PROGRAM In this section we formalize the intuitive feelings we have with regard to the relationship between the primal and dual versions of the two illustrative examples we examined The important thing to observe is that the relationship - for the standard form - is given as a definition.
  • 149. 2-149 Standard form of the Primal Problem a x + a x + ... + a x £ b a x + a x + ... + a x £ b 11 1 12 2 1 1 21 1 22 2 2 2 ... ... ... ... ... ... ... ... a x + a x + ... + a x £ b x x x n n n n m m mn n m n 1 1 2 2 ³ , ,..., 0 1 2 max x j j j n Z = å c x =1
  • 150. 2-150 Standard form of the Dual Problem min y a y + a y + ... + a y ³ c a y + a y + ... + a y ³ c 11 1 21 2 1 1 12 1 22 2 2 2 ... ... ... ... ... ... ... ... a y + a y + ... + a y ³ c y y y m m m m n n mn m n m 1 1 2 2 ³ , ,..., 0 1 2 i i m i w b y = å= 1
  • 151. 2-151 Definition Primal Problem Dual Problem z *: = max Z = cx x s . t . Ax £ b x ³ 0 =min w*:x w=yb s.t. yA³c y ³0 b is not assumed to be non-negative
  • 152. 2-152 Example Primal x x x x x x x x x - + - £ + - + + £ 3 8 9 15 20 18 5 8 4 12 30 1 2 4 5 1 2 3 4 5 0 x x x x x , , , , ³ 1 2 3 4 5 max x Z = 5x1 + 3x2 - 8x3 + 0x4 +12x5
  • 153. 2-153 w = 20y1 + 30y2 3 18 5 8 5 3 min y + ³ 1 2 1 2 - + ³ - ³ - + ³ 8 8 2 9 4 0 15 12 12 1 2 1 2 1 2 0 y y y y y y y y y y y - + ³ , ³ Dual
  • 154. 2-154 Primal-Dual relationship x1 ³ 0 x2 ³ 0 xn ³ 0 w= y1 ³ 0 a11 a12 a1n £ b1 Dual y2 ³ 0 a21 a22 a2n £ b2 (min w) ... ... ... ... ... ym ³ 0 am1 am2 amn £ bn ³ ³ ³ Z= c1 c2 cn
  • 155. 2-155 Example x x x x x x x x x x x - + £ 5 18 5 15 - 8 + 12 - 8 £ 8 12 - 4 + 8 £ 10 2 5 5 - £ , , ³ 0 1 2 3 1 2 3 1 2 3 1 3 x x x 1 2 3 max x Z = 4x1 +10x2 - 9x3
  • 156. 2-156 Conversion Multiply through the greater-than-or-equal-to inequality constraint by -1 Use the approach described above to convert the equality constraint to a pair of inequality constraints. Replace the variable unrestricted in sign, , by the difference of two nonnegative variables.
  • 157. 2-158 Dual y2 - y3 + y4 ³ 1 -2y1 - 3y2 + 3y3 - 2y4 ³ 1 y1 + 4y2 - 4y3 ³ 1 - y1 - 4y2 + 4y3 ³ -1 y1, y2, y3, y4 ³ 0 min y w = -4y1 + 5y2 - 5y3 + 3y4
  • 158. 2-159 Streamlining the conversion ... An equality constraint in the primal generates a dual variable that is unrestricted in sign. An unrestricted in sign variable in the primal generates an equality constraint in the dual.
  • 159. 2-160 (Continued) min y w = -4y1 + 5y2 - 5y3 + 3y4 y2 - y3 + y4 ³ 1 -2y1 - 3y2 + 3y3 - 2y4 ³ 1 y1 + 4y2 - 4y3 ³ 1 - y1 - 4y2 + 4y3 ³ -1 y1, y2, y3, y4 ³ 0
  • 160. 2-161 + w = -4y1 + 5y2 + 3y3 , , y y + ³ 2 3 , , , , , , , , ; , y y y y y y y y - - - ³ 1 2 3 1 2 1 3 2 1 2 3 2 1 - + = 4 1 ³ 0 urs min , , , , y + correction
  • 161. 2-162 Primal-Dual relationship Primal Problem opt=max Constraint i : = form = form Variable j: xj = 0 xj urs Dual Problem opt=min Variable i : yi = 0 yi urs Constraint j: = form = form
  • 162. 2-163 Example x x x x x x x - ³ - + = - 3 8 6 1 2 1 2 1 2 1 6 5 8 10 0 = ³ ; urs max x Z = 5x1 + 4x2
  • 163. 2-164 equivalent non-standard form max x Z = 5x1 + 4x2 - x + x £ x x x x ; x urs 3 8 6 1 2 1 2 + = - = ³ 6 5 8 10 0 1 2 1
  • 164. 2-165 Dual from the recipe y y y y y y ; y , y urs - + + = 3 8 5 8 6 4 1 2 3 1 2 1 2 3 + ³ ³ 0 min y w = 6y1 - 5y2 +10y3
  • 165. 2-166 What about opt=min ? Can use the usual trick of multiplying the objective function by -1 (remembering to undo this when the dual is constructed.) It is instructive to use this method to construct the dual of the dual of the standard form. i.e, what is the dual of the dual of the standard primal problem?
  • 166. 2-167 What is the dual of =min w*:x w=yb s.t. yA³c y³0
  • 167. 2-168 max y - w = - yb s.t. yA ³ c y ³ 0 max y - w = -yb s.t. - yA £ -c y ³ 0
  • 168. 2-169 min x . . Z cx s t - = - Ax b x - ³ - ³ 0 max x . . Z cx s t = Ax £ b x ³ 0
  • 169. 2-170 Important Observation FOR ANY PRIMAL LINEAR PROGRAM, THE DUAL OF THE DUAL IS THE PRIMAL
  • 170. 2-171 Primal-Dual Relationship Primal or Dual opt=max opt=min Dual or Primal Variable i : yi = 0 yi urs Constraint j: = form = form Constraint i : = form = form Variable j: xj = 0 xj urs
  • 171. 2-172 Example min x Z = 6x1 + 4x2 x x x x x x x x - ³ + = - - £ , ³ 3 5 12 2 8 5 10 0 1 2 1 2 1 2 1 1
  • 172. 2-173 equivalent form min x Z = 6x1 + 4x2 3 5 12 1 2 1 2 2 8 5 10 1 2 1 2 0 x x x x x x x x - ³ + = - - + ³ - , ³
  • 173. 2-174 Dual y y y y y y y y y + - £ 3 5 6 5 2 4 1 2 3 1 2 3 1 3 2 - + + £ , ³ 0 ; urs max y w =12y1 - 8y2 - 10y3
  • 174. 2-175 Exercise Maximize z = 3 x + 9 y such that x + 4 y ≤ 8 x + 2 y ≤4 x, y≥0 Determine its dual ,Solve both the problems and compare the results